You are on page 1of 85

XI MODULE - V

BINOMIAL THEOREM AND


PERMUTATION & COMBINATION

Modus Operandi for Practice


(Suggested Ideal Approach)

Step – I CatalyseR 's Practice Sheets & NCERT Problems


{while Chapter is running in Class-Room}
(Mandatory)

Step – II
CatalyseR 's Module
(Mandatory)

Step – III Previous Years’ KVPY &


JEE Subjective & Objective Questions
(Mandatory)

Step – IV After Completion of Step – III, if time permits, students can


solve questions from Mathematics books by
(Optional) R. D. Sharma (Objective), A. Das Gupta (Subjective)

This Study Package is Prepared by


d`fr R
atalyse
n t W ing of C
Conte
BASIC MATHEMATICS 1

BINOMIAL THEOREM AND


PERMUTATION & COMBINATION
INDEX
 CONCEPTS IN BRIEF (BINOMIAL THEOREM) 01 – 09
 SOLVED EXAMPLES 10 – 12
 ADVANCED OBJECTIVE EXERCISES # 01 13 – 17
 ADVANCED OBJECTIVE EXERCISES # 02 18 – 23
 ADVANCED SUBJECTIVE EXERCISES # 01 23 – 26
 ADVANCED SUBJECTIVE EXERCISES # 02 27 – 28
 CONCEPTS IN BRIEF (PERMUTATION & COMBINATION) 29 – 39
 SOLVED EXAMPLES 40 – 42
 ADVANCED OBJECTIVE EXERCISES # 01 to # 10 43 – 68
 ADVANCED SUBJECTIVE EXERCISES # 01 69 – 72
 ADVANCED SUBJECTIVE EXERCISES # 02 73 – 75
 JEE (MAIN) CORNER 76 – 79
 JEE (ADVANCED) CORNER 80 – 82

WEIGHTAGE OF ‘BINOMIAL THEOREM AND PERMUTATION & COMBINATION’


IN JEE (MAIN & ADVANCED) in Last Three Years
JEE (MAIN) Formely known as AIEEE

MARK /
YEAR No. Of Qs.
MATHEMATICS TOTAL MARKS
2015 2 8/120
2016 2 8/120
2017 2 8/120

JEE (ADVANCED)
MARK /
YEAR No. Of Qs.
MATHEMATICS TOTAL MARKS
2015 2 8/168
2016 2 6/124
2017 2 6/120

Nothing is impossible, the word itself says ‘I’m possible’!


BINOMIAL THEOREM AND PERMUTATION & COMBINATION 1

BINOMIAL THEOREM
1.1. BINOMIAL THEOREM THEORY
1
1.1.1. Binomial Expression
An algebraic expression consisting of two terms with +ve or – ve sign between them is
called a binomial expression.
 p q  1 4 
For example : (a  b), (2x  3y),  2
 4  ,   3  etc.
x x  x y 

1.1.2. Binomial Theorem for Positive Integral Index


The rule by which any power of binomial can be expanded is called the binomial
theorem. If n is a positive integer and x, y  C then
(x  y)n nC0 xn 0 y 0 nC1xn1y1  nC2 xn 2 y 2  ........ nCr xnr yr
...... nCn1xyn1 nCn x0 yn
n
i.e., (x  y)   nCr .x n r .yr
n
.....(i)
r 0

Here n C0 , n C1 , n C2 ,......n Cn are called binomial coefficients


n!
and n C r  for 0  r  n .
r ! (n  r ) !
IMPORTANT TIPS

 The number of terms in the expansion of (x  y)n are (n + 1).


 The expansion contains decreasing power of x and increasing power of y. The sum of the
powers of x and y in each term is equal to n.
 The binomial coefficients n
C0 , n C1 , n C2 ........ equidistant from beginning and end are equal
n
i.e., Cr  nCn  r .
 ( x  y)n  Sum of odd terms + sum of even terms.

1.1.3. Some Important Expansions


(1) Replacing y by – y in (i), we get,
(x  y)n  n C0 x n0 .y 0  n C1 x n1 .y 1  n C2 x n 2 . y 2 ....  (1)r n Cr x nr .y r  ....  (1)n n Cn x 0 .y
n
i.e., (x  y)n   (1) r n C r x nr .y r .....(ii)
r 0

The terms in the expansion of ( x  y)n are alternatively positive and negative, the last
term is positive or negative according as n is even or odd.

Nothing is impossible, the word itself says ‘I’m possible’!


2 BINOMIAL THEOREM AND PERMUTATION & COMBINATION

(2) Replacing x by 1 and y by x in equation (i) we get,


(1  x)n  n C0 x 0  n C1 x 1  n C 2 x 2  ......  n Cr x r  ......  n Cn x n
n
i.e., (1  x)n   nCr x r
r 0

This is expansion of (1  x)n in ascending power of x.

(3) Replacing x by 1 and y by – x in (i) we get,


(1  x)n  n C0 x 0  n C1 x 1  n C2 x 2  ......  (1)r n Cr x r  ....  (1)n n Cn x n
n
i.e., (1  x)n   (1)r n Cr x r
r 0

(4) (x  y)n  (x  y)n  2[ n C0 x n y 0  n C 2 x n 2 y 2  n C4 x n 4 y 4  .......] and


(x  y)n  (x  y)n  2[ n C1 x n1 y 1  n C3 x n 3 y 3  n C5 x n5 y 5  .......]

(5) The coefficient of (r  1)th term in the expansion of (1  x)n is nCr .

(6) The coefficient of x r in the expansion of (1  x)n is nCr .

Note :  If n is odd, then (x  y)n  (x  y)n and (x  y)n  (x  y)n , both have the same
 n  1
number of terms equal to  .
 2 

n n
 If n is even, then ( x  y)n  ( x  y)n has   1  terms and (x  y)n  (x  y)n has
2  2
terms.

1.1.4. General Term

(x  y)n  n C0 x n y 0  n C1 x n1 y 1  n C 2 x n 2 y 2  .....  n Cr x nr y r  ....  n Cn x 0 y n


Let Tr  1 denote the (r + 1)th term  Tr 1  n Cr x nr y r
This is called general term, because by giving different values to r, we can determine all
terms of the expansion.
In the binomial expansion of (x  y) n , Tr 1  (1)r n Cr x nr y r
In the binomial expansion of (1  x)n , Tr 1 nCr x r
In the binomial expansion of (1  x)n , Tr 1  (1)r nCr x r

Note :  In the binomial expansion of (x  y)n , the pth term from the end is (n  p  2)th
term from beginning.

CatalyseR Eduventures (India) Pvt. Ltd.


BINOMIAL THEOREM AND PERMUTATION & COMBINATION 3

IMPORTANT TIPS

Tr 1  n  r  1  y
 In the expansion of ( x  y)n , n  N  
Tr  r x
n(n  1)
 The coefficient of x n1 in the expansion of ( x  1)( x  2).......( x  n)  
2
n(n  1)
 The coefficient of x n1 in the expansion of ( x  1)( x  2).....( x  n) 
2

1.1.5. Number of Terms in the Expansion of (a  b  c) n and (a  b  c  d )n

(a  b  c )n can be expanded as : (a  b  c ) n  {(a  b)  c} n

 (a  b)n  n C1 (a  b)n1 (c)1  n C 2 (a  b)n 2 (c) 2  .....  n Cn c n


 (n  1) term  n term  (n  1)term  ...  1term
(n  1)(n  2)
 Total number of terms = (n  1)  (n)  (n  1)  ......  1  .
2
(n  1)(n  2)(n  3)
Similarly, Number of terms in the expansion of (a  b  c  d)n  .
6

1.1.6. Middle Term

The middle term depends upon the value of n.

(1) When n is even, then total number of terms in the expansion of ( x  y)n is n  1 (odd).
So there is only one middle term
th
n 
i.e.,   1 term is the middle term.
2 
T n 
 n Cn / 2 x n / 2 y n / 2
 2 1 
 

(2) When n is odd, then total number of terms in the expansion of ( x  y)n is n  1 (even). So,
th th
 n 1 n 3
there are two middle terms i.e.,   and   are two middle terms.
 2   2 
n1 n1 n1 n1
n n
T n1   C n1 x 2 y 2 and T n 3   C n1 x 2 y 2
   
 2  2  2  2

Note :  When there are two middle terms in the expansion then their binomial coefficients
are equal.
 Binomial coefficient of middle term is the greatest binomial coefficient.

Nothing is impossible, the word itself says ‘I’m possible’!


4 BINOMIAL THEOREM AND PERMUTATION & COMBINATION

1.1.7. Determine a Particular Term in the Expansion


n
 1  m
In the expansion of  x    , if x occurs in Tr 1 ,
 x 
n  m
then r is given by n  r(   )  m  r 

Thus in above expansion if constant term which is independent of x, occurs in Tr 1 then r
n
is determined by n  r(   )  0  r 


1.1.8. Greatest Term & Greatest Coefficient

(1) Greatest term : If Tr and Tr 1 be the rth and (r  1) th terms in the expansion of
n
Tr 1 C xr nr 1
(1  x) n , then  n r r 1  x
Tr C r 1 x r
Let numerically, Tr 1 be the greatest term in the above expansion.
Tr  1
Then Tr 1  Tr or 1
Tr
n r 1
 | x | 1 or r  (n  1) | x | ......(i)
r (1 | x |)

Now substituting values of n and x in (i), we get r  m  f or r  m

where m is a positive integer and f is a fraction such that 0  f  1 .


When n is even Tm1 is the greatest term, when n is odd Tm and Tm1 are the greatest terms
and both are equal.
Short cut method : To find the greatest term (numerically) in the expansion of (1  x)n .
x (n  1)
(i) Calculate m =
x 1
(ii) If m is integer, then Tm and Tm1 are equal and both are greatest term.
(iii) If m is not integer, there T[m]1 is the greatest term, where [.] denotes
the greatest integral part.

(2) Greatest coefficient


(i) If n is even, then greatest coefficient is n C n / 2

(ii) If n is odd, then greatest coefficient are n C n1 and n C n 3


2 2

CatalyseR Eduventures (India) Pvt. Ltd.


BINOMIAL THEOREM AND PERMUTATION & COMBINATION 5

IMPORTANT TIPS

 For finding the greatest term in the expansion of ( x  y)n .


n
 y
we rewrite the expansion in this form ( x  y)n  x n 1   .
 x
n
 y
Greatest term in (x + y)n  x n . Greatest term in 1  
 x

1.1.9. Properties of Binomial Coefficients

In the binomial expansion of (1  x)n ,

(1  x) n  n C 0  n C1 x  n C 2 x 2  .....  n C r x r  ....  n Cn x n .
where n C0 , n C1 , n C 2 ,......, n Cn are the coefficients of various powers of x and called
binomial coefficients, and they are written as C0 , C1 , C2 , .....Cn .
Hence, (1  x)n  C0  C1 x  C2 x 2  .....  Cr x r  .....  Cn x n .....(i)
(1) The sum of binomial coefficients in the expansion of (1  x)n is 2n .
Putting x  1 in (i), we get 2n  C0  C1  C 2  .....  Cn .....(ii)
(2) Sum of binomial coefficients with alternate signs : Putting x  1 in (i)
We get, 0  C0  C1  C2  C3  ...... …..(iii)

(3) Sum of the coefficients of the odd terms in the expansion of (1  x)n is equal to
sum of the coefficients of even terms and each is equal to 2n1 .
From (iii), we have C0  C 2  C4  .....  C1  C 3  C5  ........ ......(iv)
i.e., sum of coefficients of even and odd terms are equal.
From (ii) and (iv), C0  C2  C4  .....  C1  C3  C5  .....  2n1 ......(v)
n n n 1 n n  1 n 2
(4) Cr  Cr 1  . Cr  2 and so on.
r r r 1
1
(5) Sum of product of coefficients : Replacing x by in (i) we get
x
n
 1 C C C
 1    C0  1  22  ... nn  .... (vi)
 x x x x
Multiplying (i) by (vi), we get
(1  x )2 n  C C 
n
 (C0  C1 x  C 2 x 2  ......)  C0  1  22  ..... 
x  x x 
Now comparing coefficient of x r on both sides.
We get, 2nCn  r  C0 Cr  C1Cr 1  ......Cn  r .Cn .....(vii)

(6) Sum of squares of coefficients : Putting r  0 in (vii), we get


2n
Cn  C02  C12  ......Cn2
n
(7) Cr nCr 1 n 1Cr

Nothing is impossible, the word itself says ‘I’m possible’!


6 BINOMIAL THEOREM AND PERMUTATION & COMBINATION

(1) Use of Differentiation : This method applied only when the numericals occur as the
product of binomial coefficients.

(2) Use of Integration : This method is applied only when the numericals occur as
the denominator of the binomial coefficients.
Solution process : If (1  x)n  C0  C1 x  C2 x 2  .....  Cn x n , then we integrate both
sides between the suitable limits which gives the required series.
(i) If the sum contains C0 , C1 , C2 ,....... Cn with all positive signs, then integrate
between limit 0 to 1.
(ii) If the sum contains alternate signs (i.e. +, –) then integrate between limit –
1 to 0.
(iii) If the sum contains odd coefficients i.e., (C0, C2, C4.....)then integrate
between –1 to 1.
(iv) If the sum contains even coefficients (i.e., C1 , C3 , C5 .....) then subtracting (ii) from
(i) and then dividing by 2.
(v) If in denominator of binomial coefficients is product of two numericals then
integrate two times, first taking limit between 0 to x and second time take suitable
limits.

1.1.10. An Important Theorem

If ( A  B)n = I  f where I and n are positive integers, n being odd and 0  f  1 then
(I  f ). f  K n where A  B 2 = K  0 and A  B  1.

Note :  If n is even integer then ( A  B)n  ( A  B) n  I  f  f 

Hence L.H.S. and I are integers.


 f  f is also integer;  f  f   1 ;  f   (1  f )

Hence (I  f ) (1  f )  (I  f ) f  = ( A  B) n ( A  B) n = ( A  B 2 ) n = K n .

1.1.11. Multinomial Theorem (For Positive Integral Index)

If n is positive integer and a1 , a 2 , a 3 , ....a n  C


n!
then (a1  a 2  a 3  ...  a m )n   a n1 a n2 ...a mnm
n1 ! n 2 ! n 3 !...nm ! 1 2
Where n1 , n 2 , n3 ,.....nm are all non-negative integers subject to the condition,
n1  n2  n3  .....nm  n .

CatalyseR Eduventures (India) Pvt. Ltd.


BINOMIAL THEOREM AND PERMUTATION & COMBINATION 7

(1) The coefficient of a 1n .a n2 ..... a mn in the expansion of (a1  a2  a3  ....am )n is


1 2 m

n!
n1 ! n2 ! n3 !.... nm !
n!
(2) The greatest coefficient in the expansion of (a1  a 2  a 3  ....am )n is mr
(q! ) [(q  1)! ]r
Where q is the quotient and r is the remainder when n is divided by m.
(3) If n is +ve integer and a1 , a 2 ,.....a m  C, a 1n . a n2 ......... a mn then coefficient of x r in the
1 2 m

n!
expansion of (a1  a2 x  .....am x m1)n is  n ! n !n !.....n
1 2 3 m!

Where n1 , n2 .....nm are all non-negative integers subject to the condition:


n1  n2  .....nm  n and n2  2n3  3n4  ....  (m  1)nm  r .
(4) The number of distinct or dissimilar terms in the multinomial expansion
(a1  a 2  a 3  ....am )n is n m1
Cm1 .

1.1.12. Binomial Theorem for any Index

(1) Statement:
n(n  1)x2 n(n  1) (n  2) 3 n(n  1) ......(n  r  1) r
(1  x)n  1  nx   x  ....  x  ...termsup to 
2! 3! r!
When n is a negative integer or a fraction, where  1  x  1 , otherwise expansion will
not be possible.If x  1, the terms of the above expansion go on decreasing and if x be
very small a stage may be reached when we may neglect the terms containing higher
power of x in the expansion, then (1  x )n  1  nx .

IMPORTANT TIPS

 Expansion is valid only when 1  x  1 .


 n
Cr can not be used because it is defined only for natural number, so n Cr will be written
(n) (n  1)......(n  r  1)
as
r!
 The number of terms in the series is infinite.
n
 y
 If first term is not 1, then make first term unity in the following way, ( x  y) n  x n 1   ,
 x
y
if  1.
x

Nothing is impossible, the word itself says ‘I’m possible’!


8 BINOMIAL THEOREM AND PERMUTATION & COMBINATION

n(n  1)(n  2)......( n  r  1) r


(2) General term : Tr 1  x
r!
Some important expansions:
n(n  1) 2 n(n  1)(n  2)......( n  r  1) r
(i) (1  x)n  1  nx  x  .......  x  ......
2! r!
n(n  1) 2 n(n  1)(n  2).....( n  r  1)
(ii) (1  x)n  1  nx  x  .......  ( x )r  .......
2! r!
n n(n  1) 2 n(n  1) (n  2) 3 n(n  1)......( n  r  1) r
(iii) (1  x)  1  nx  x  x  .....  x  .....
2! 3! r!
n(n  1) 2 n(n  1)(n  2) 3 n(n  1)......(n  r  1)
(iv) (1  x)n  1  nx  x  x  .....  ( x)r  ......
2! 3! r!
(a) Replace n by 1 in (iii) : (1  x ) 1  1  x  x 2  .....  x r  ......  ,
General term, Tr 1  x r
(b) Replace n by 1 in (iv) : (1  x ) 1  1  x  x 2  x 3  .....  ( x ) r  ......  ,
General term, Tr 1  ( x)r .
(c) Replace n by 2 in (iii) : (1  x ) 2  1  2 x  3 x 2  .....  (r  1) x r  .....  ,
General term, Tr 1  (r  1)x r .
(d) Replace n by 2 in (iv) :
(1  x)2  1  2x  3 x 2  4 x 3  ......  (r  1) ( x)r  .....
General term, Tr 1  (r  1) (x)r .
(e) Replace n by 3 in (iii) :
(r  1) (r  2) r
(1  x )  3  1  3 x  6 x 2  10 x 3  .....  x
2!
 ..........  General term, Tr 1  (r  1) (r  2) / 2! . x r
(f) Replace n by 3 in (iv)
(r  1) (r  2)
(1  x) 3  1  3 x  6 x 2  10 x 3  .....  ( x )r  ..... 
2!
(r  1)(r  2)
General term, Tr 1  ( x )r
2!

1.1.13. Three/Four Consecutive Terms or Coefficients

(1) If consecutive coefficients are given: In this case divide consecutive coefficients pair
wise. We get equations and then solve them.
(2) If consecutive terms are given : In this case divide consecutive terms pair wise i.e. if four
Tr Tr 1 Tr  2
consecutive terms be Tr , Tr 1 , Tr  2 , Tr  3 then find , ,  1 ,  2 ,  3 (say) then
Tr 1 Tr  2 Tr  3
divide  1 by 2 and 2 by 3 and solve.

CatalyseR Eduventures (India) Pvt. Ltd.


BINOMIAL THEOREM AND PERMUTATION & COMBINATION 9

1.1.14. Some Important Points

(1) Pascal's Triangle :


1 (x  y)0

1 1 ( x  y)1
1 2 1 ( x  y)2
1 3 3 1 (x  y)3
1 4 6 4 1 (x  y)4
1 5 10 10 5 1 ( x  y)5
Pascal's triangle gives the direct binomial coefficients.
Example : (x  y) 4  1x 4  4 x 3 y  6 x 2 y 2  4 xy 3  y 4

(2) Method for finding terms free from radical or rational terms in the expansion of
(a1 / p  b1 / q )N  a, b  prime numbers : Find the general term
N r r
N 1 / p N r 1/ q r N p q
Tr 1  Cr (a ) (b )  Cr a .b
Putting the values of 0  r  N , when indices of a and b are integers.

Note :  Number of irrational terms = Total terms – Number of rational terms.

Nothing is impossible, the word itself says ‘I’m possible’!


10 BINOMIAL THEOREM AND PERMUTATION & COMBINATION

1.2. SOLVED EXAMPLES:


1. If roots of the equation  m

C0  m C1 .......m Cm x 2 -  n

C0  n C 2  n C4 ....... x +

 n

C1  n C3  n C5 ....... =0 are real , find minimum value of n – m.
(a) 1 (b) 2 (c) 3 (d) –1
Solution: (c) Roots are real if 2 n1 2
 4( 2m )2n1  0
2 2n  2  2 m n 1  0 2n – 2  m+ n +1 n–m3
minimum value of n – m = 3
2. A number is said to be a nice number if it has exactly 4 factors. (Including one
and number itself). Let n = 23  32  53  7  112 , then number of factors, which
are nice numbers is
(a) 36 (b) 12 (c) 10 (d) 147
Solution: (b) Any number having exactly 4 factors is of the form m = p3 (p prime) or m =
p.q (where p & q are distinct primes) So we have 5C2 + 2 = 12 such factors.

3. The number of terms in the expansion of


n
(1 + x)(1 + x3)( 1+ x6)( 1+x12) (1+x24) . . . . (1 + x32 ) is
(a) 2n+3 (b) 2n+4 (c) 2n+5 (d) none of these
Solution: (d) After expansion, no two terms will have the same powers of x or the
terms are non over- lapping.
Therefore, the total number of terms = 2  2  2  . . . (n +2) times = 2n+2 as a
particular power of x can be chosen from each bracket in 2 ways.

4. If coefficient of x2 y3 z4 in (x + y +z)n is A, then coefficient of x4 y4 z is


nA A
(a) 2A (b) (c) (d) none of these
2 2
Solution: (c) Since x2 y3z4 is occurring in the expansion of (x +y +z)n, so n should be 9
9! 9!
only. Now A = = 1260 Coefficient of x4 y4 z is = 630 = A/2.
2!  3!  4! 4!  4 !

T2 T
5. If in the expansion of (a + b)n and 3 in the expansion of (a b)n + 3 are equal,
T3 T4
then n is equal to
(a) 3 (b) 4
(c) 5 (d) 6

CatalyseR Eduventures (India) Pvt. Ltd.


BINOMIAL THEOREM AND PERMUTATION & COMBINATION 11

(n  3)(n  2)
n
C1an 1b n 3
C2an 1b2 n 2
Solution: n
  
C2an  2b2 n3
C3anb3 n(n  1) (n  3 )(n  2)(n  1)
2 6
2 3
   2n + 2 = 3n –3  n = 5
n 1 n 1
Hence (c) is the correct answer.

6. Coefficient of x5 in the expansion of (1 + x2)5(1 + x)4 is


(a) 61 (b) 59
(c) 0 (d) 60
Solution: (1 + x ) (1 + x) = (1 + 5x + 10x + …..)(1 + x)4
2 5 4 2 4

 Coefficient of x5 = 5  4C3 + 10 4C1 = 20 + 40 = 60.


Hence (d) is the correct answer.

7.
n
The coefficient of x in 1  x 
x2 x3
  .... 
 1n x n 
 is
2! 3! n! 
 

(a)
 nn (b)
 2n (c) 1
(d) – 1 2
2
n! n! n! n!
2
 x2 x3 ( 1)n xn 
Solution: Coefficient of xn in  1  x    ...  

 2! 3! n! 
2
n x 2 x3 
Coefficient of x in  1  x    ... 
 2! 3 ! 
Coefficient of xn in (ex)2
( 2)n
Coefficient of xn in e2x =
n!

8. If x > –1, then the statement P(n) : (1 + x)n > 1 + nx is true for
(a) all n  N (b) all n > 1
(c) all n > a and x  0 (d) None of these.
Solution: P(1) is not true.
For n = 2, P(2) : (1 + x)2 > 1 + 2x is true if x  0
Let P(k) : (1 + x)k > 1 + kx be true
 (1 + x)k+1 = (1 + x) (1 + x)k > (1 + x) (1 + kx)
= 1 + (k + 1)x + kx2
> 1 + (k + 1)x ( kx2 > 0)
Hence (c) is the correct answer.

Nothing is impossible, the word itself says ‘I’m possible’!


12 BINOMIAL THEOREM AND PERMUTATION & COMBINATION

9. The smallest positive integer for which the statement 3n+1 < 4n holds is
(a) 1 (b) 2 (c) 3 (d) 4
Solution: The given statement is true for n  4.
Hence (d) is the correct answer.

1
10. If the binomial expansion of (a + bx)- 2 is - 3x + ...... , where a > 0, then (a, b) is
4
(a) (2, 12) (b) (2, 8) (c) (–2, 12) (d) None of these.
2
 b 
Solution: (a  bx)2  a 2  1  x 
 a 
1  b   1 2b

a2 1  ( 2)  a x   ......  a2  a3 x  .....
   
1
Also, (a + bx)- 2 = - 3x + .....
4
1 1 2b
  . . . (1) and - =-3 . . . (2)
a2 4 a3
(1)  a2  4  a  2 and from (2) b = 12
Hence (a) is the correct answer.

11. The term independent of x in the expansion of (1+ x)n (1 + 1/ x)n is


(a) C 0 2 + 2C12 + 3C 2 2 + ...... + (n + 1)C n 2 (b) (C 0 + C1 + ...... + C n )2
(c) C 0 2 + C12 + ...... + C n 2 (d) None of these.
Solution: We have,
n
 1  C C C 
(1  x)  1    (C0  C1x  C2 x 2  ......  Cn x n )   C0  1  22  ......  nn 
n
 x  x x x 
Term independent of x on the R.H.S. = C 0 2 + C12 + C 2 2 + ...... + C n 2 .
Hence (c) is the correct answer.



CatalyseR Eduventures (India) Pvt. Ltd.


BINOMIAL THEOREM AND PERMUTATION & COMBINATION 13

1.3. ADVANCE OBJECTIVE EXERCISES:

ADVANCE OBJECTIVE EXERCISE - 1


Q.1 Given that the term of the expansion (x1/3  x1/2)15 which does not contain x is 5 m
where m  N , then m =
(a) 1100 (b) 1010 (c) 1001 (d) none

Q.2 In the binomial (21/3 + 31/3)n, if the ratio of the seventh term from the beginning of
the expansion to the seventh term from its end is 1/6 , then n =
(a) 6 (b) 9 (c) 12 (d) 15
n
 x
Q.3 If the coefficients of x7 & x8 in the expansion of  2   are equal , then the value
 3
of n is :
(a) 15 (b) 45 (c) 55 (d) 56

 1 1   1 
Q.4 The coefficient of x49 in the expansion of (x – 1)  x    x  2  .....  x  49  is equal
 2 2   2 
to
 1 
(a) – 2  1  50  (b) + ve coefficient of x
 2 
 1 
(c) – ve coefficient of x (d) – 2  1  49 
 2 
Q.5 The last digit of (3P + 2) is :
(a) 1 (b) 2 (c) 4 (d) 5
where P = 34n and n  N
n
 1
Q.6 The sum of the binomial coefficients of  2 x   is equal to 256 . The constant
 x
term in the expansion is
(a) 1120 (b) 2110 (c) 1210 (d) none
10
x 3
Q.7 The coefficient of x4 in   2  is :
2 x 
405 504 450 405
(a) (b) (c) (d)
256 259 263 512

Q.8 The remainder, when (1523 + 2323) is divided by 19, is


(a) 4 (b) 15 (c) 0 (d) 18

Nothing is impossible, the word itself says ‘I’m possible’!


14 BINOMIAL THEOREM AND PERMUTATION & COMBINATION

Q.9 Let (7  4 3 ) n = p +  when n and p are positive integers and   (0, 1)


then (1 – ) (p + ) is
(a) rational which is not an integer (b) a prime
(c) a composite (d) none of these

Q.10 If (11)27 + (21)27 when divided by 16 leaves the remainder


(a) 0 (b) 1 (c) 2 (d) 14

Q.11 Last three digits of the number N = 7100 – 3100 are


(a) 100 (b) 300 (c) 500 (d) 000

Q.12 The last two digits of the number 3400 are :


(a) 81 (b) 43 (c) 29 (d) 01

Q.13 If (1 + x + x²)25 = a0 + a1x + a2x² + ..... + a50 . x50 then a0 + a2 + a4 + ..... + a50 is :
(a) even (b) odd & of the form 3n
(c) odd & of the form (3n  1) (d) odd & of the form (3n + 1)

Q.14 The sum of the series (1² + 1).1! + (2² + 1).2! + (3² + 1). 3! + ..... + (n² + 1). n! is :
(a) (n + 1). (n+2)! (b) n.(n+1)!
(c) (n + 1). (n+1)! (d) none of these

Q.15 Let Pm stand for nPm . Then the expression 1 . P1 + 2 . P2 + 3 . P3 + ..... + n . Pn =


(a) (n + 1) !  1 (b) (n + 1) ! + 1 (c) (n + 1) !  (d) none of these

1  1  4 x  1  7 1  4 x  1  7 
Q.16 The expression      is a polynomial in x of
4x 1   2   2  

degree
(a) 7 (b) 5 (c) 4 (d) 3
n
n
 a  C3
Q.17 If the second term of the expansion  a1/13   is 14a5/2 then the value of
n
is :
 a 1  C2
(a) 4 (b) 3 (c) 12 (d) 6

Q.18 If (1 + x) (1 + x + x2) (1 + x + x2 + x3) ...... (1 + x + x2 + x3 + ...... + xn)


m
 a0 + a1x + a2x2 + a3x3 + ...... + amxm then a
r 0
r has the value equal to

(a) n! (b) (n + 1) !
(c) (n – 1)! (d) none

CatalyseR Eduventures (India) Pvt. Ltd.


BINOMIAL THEOREM AND PERMUTATION & COMBINATION 15

Q.19 The value of 4 {nC1 + 4 . nC2 + 42 . nC3 + ...... + 4n  1} is :


(a) 0 (b) 5n + 1 (c) 5n (d) 5n  1

Q.20 If n be a positive integer such that n  3, then the value of the sum to n terms of the series

1.n
 n  1 (n  1) +  n 1  n  2  (n  2) –
 n 1  n  2   n  3 (n  3) + ...... is :
1! 2! 3!
(a) 0 (b) 1 (c) – 1 (d) none of these

Q.21 In the expansion of (1 + x)43 if the coefficients of the (2r + 1)th and the (r + 2)th terms
are equal, the value of r is :
(a) 12 (b) 13 (c) 14 (d) 15

Q.22 The positive value of a so that the coefficient of x5 is equal to that of x15 in the
10
 a 
expansion of  x 2  3  is
 x 
1 1
(a) (b) (c) 1 (d) 2 3
2 3 3

10
 x 1 x 1 
Q.23 In the expansion of  2/3 1/3
 1/2 
, the term which does not contain x is :
x  x 1 x  x 
(a) 10C0 (b) 10C7 (c) 10C4 (d) none

8
 1 
Q.24 If the 6th term in the expansion of the binomial  8/3  x 2 log10 x  is 5600, then x equals
x 
to
(a) 5 (b) 8 (c) 10 (d) 100

Q.25 Co-efficient of t in the expansion of,


( + p)m  1 + ( + p)m  2 ( + q) + ( + p)m  3 ( + q)2 + ...... ( + q)m  1
where  q and p  q is :

(a)
m

Ct pt  q t  (b)
m

Ct pm  t  qm  t  (c) m

Ct pt  qt  (d)
m

Ct pm  t  qm t 
pq p q p q pq

Q.26 (1 + x) (1 + x + x2) (1 + x + x2 + x3) ...... (1 + x + x2 + ...... + x100) when written in the


ascending power of x then the highest exponent of x is ______ .
(a) 4950 (b) 5050 (c) 5150 (d) none

Nothing is impossible, the word itself says ‘I’m possible’!


16 BINOMIAL THEOREM AND PERMUTATION & COMBINATION

n
Q.27 Let 5  2 6  = p + f where n  N and p  N and 0 < f < 1 then the value of,

f2  f + pf  p is
(a) a natural number (b) a negative integer
(c) a prime number (d) are irrational number
100
Q.28 Number of rational terms in the expansion of  2  43  is :

(a) 25 (b) 26 (c) 27 (d) 28


10
 cos  
Q.29 The greatest value of the term independent of x in the expansion of  x sin    is
 x 
10
10C C5
(a) 5 (b) 25 (c) 25 · 10C5 (d)
25

Q.30 If (1 + x – 3x2)2145 = a0 + a1x + a2x2 + ......... then a0 – a1 + a2 – a3 + ..... ends with


(a) 1 (b) 3 (c) 7 (d) 9
9
 4 x2 3 
Q.31 Coefficient of x6 in the binomial expansion    is
 3 2x 
(a) 2438 (b) 2688 (c) 2868 (d) none
18
 1 
Q.32 The term independent of ' x ' in the expansion of  9 x   , x > 0 , is  times
 3 x
the corresponding binomial co-efficient . Then '  ' is :
1 1
(a) 3 (b) (c)  (d) 1
3 3

Q.33 The expression [x + (x31)1/2]5 + [x  (x31)1/2]5 is a polynomial of degree :


(a) 5 (b) 6 (c) 7 (d) 8

Q.34 Given (1 – 2x + 5x2 – 10x3) (1 + x)n = 1 + a1x + a2x2 + .... and that a12 = 2a2 then the
value of n is
(a) 6 (b) 2 (c) 5 (d) 3

Q.35 The sum of the series aC0 + (a + b)C1 + (a + 2b)C2 + ..... + (a + nb)Cn is
where Cr's denotes combinatorial coefficient in the expansion of (1 + x)n, n  N
(a) (a + 2nb)2n (b) (2a + nb)2n
(c) (a +nb)2n – 1 (d) (2a + nb)2n – 1

CatalyseR Eduventures (India) Pvt. Ltd.


BINOMIAL THEOREM AND PERMUTATION & COMBINATION 17

Q.36 The coefficient of the middle term in the binomial expansion in powers of x of (1 + x)4
and of (1 – x)6 is the same if  equals
5 10 3 3
(a) – (b) (c) – (d)
3 3 10 5

Q.37 (2n + 1) (2n + 3) (2n + 5) ....... (4n  1) is equal to :


(4 n) ! (4 n) ! n ! (4 n) ! n ! (4n) ! n !
(a) n
(b) n (c) (d)
2 . (2 n) ! (2 n) ! 2 . (2 n) ! (2 n) ! (2 n) ! (2 n) ! 2 n ! (2n) !

n n
1 r T
Q.38 If Sn =  n and Tn =  n then n is equal to
r 0 Cr r 0 Cr Sn
n n 2n  1
(a) (b) 1 (c) n – 1 (d)
2 2 2

Q.39 The coefficient of xr (0  r  n  1) in the expression :


(x + 2)n1 + (x + 2)n2. (x + 1) + (x + 2)n3 . (x + 1)² + ...... + (x + 1)n1 is :
(a) nCr (2r  1) (b) nCr (2nr  1) (c) nCr (2r + 1) (d) nCr (2nr + 1)

n

Q.40 If 9  80  = 1 + f where I , n are integers and 0 < f < 1 , then :

(a) I is an odd integer (b) I is an even integer


n
(c) (I + f) (1  f) = 1 
(d) 1  f = 9  80 

ANSWER KEY

1 2 3 4 5 6 7 8 9 10 11 12 13 14
C B C A D A A C D A D D A B
15 16 17 18 19 20 21 22 23 24 25 26 27 28
A D A B D A C A C C B B B B
29 30 31 32 33 34 35 36 37 38 39 40
D B B D C A D C B A B ABD

Nothing is impossible, the word itself says ‘I’m possible’!


18 BINOMIAL THEOREM AND PERMUTATION & COMBINATION

ADVANCE OBJECTIVE EXERCISE – 2

Q.1 Given that the term of the expansion (x1/3  x1/2)15 which does not contain x is 5 m
where m  N , then m =
(a) 1100 (b) 1010 (c) 1001 (d) none

 1 1   1 
Q.2 The coefficient of x49 in the expansion of (x – 1)  x    x  2  .....  x  49  is equal
 2 2   2 
to
 1 
(a) – 2  1  50  (b) + ve coefficient of x
 2 
 1 
(c) – ve coefficient of x (d) – 2  1  49 
 2 
n
 x
Q.3 If the coefficients of x7 & x8 in the expansion of  2   are equal , then the value
 3
of n is :
(a) 15 (b) 45 (c) 55 (d) 56

Q.4 If 683 + 883 is divided by 49, then the remainder is


(a) 35 (b) 5 (c) 1 (d) 0
n
 1
Q.5 The sum of the binomial coefficients of  2 x   is equal to 256 . The constant
 x
term in the expansion is
(a) 1120 (b) 2110 (c) 1210 (d) none

C0 C1 C2 C
Q.6    ......  10 =
1 2 3 11
211 211  1 311 311  1
(a) (b) (c) (d)
11 11 11 11
Q.7 The remainder, when (1523 + 2323) is divided by 19, is
(a) 4 (b) 15 (c) 0 (d) 18
1 1 1 1
Q.8 If n  N & n is even , then    ......  =
1. (n  1) ! 3 ! ( n  3) ! 5 ! ( n  5) ! ( n  1) ! 1 !
2n 1
(a) 2n (b) (c) 2n n ! (d) none of these
n!

CatalyseR Eduventures (India) Pvt. Ltd.


BINOMIAL THEOREM AND PERMUTATION & COMBINATION 19

n
 x 5x

Q.9 In the expansion of  3 4  3 4  the sum of the binomial coefficients is 64 and the term
 
with the greatest binomial coefficient exceeds the third by (n – 1), then the value of x
must be
(a) 1 (b) 2 (c) 0 (d) – 1

Q.10 If (11)27 + (21)27 when divided by 16 leaves the remainder


(a) 0 (b) 1 (c) 2 (d) 14

Q.11 Last three digits of the number N = 7100 – 3100 are


(a) 100 (b) 300 (c) 500 (d) 000

Q.12 The last two digits of the number 3400 are :


(a) 81 (b) 43 (c) 29 (d) 01

Q.13 If (1 + x + x²)25 = a0 + a1x + a2x² + ..... + a50 . x50 then a0 + a2 + a4 + ..... + a50 is :
(a) even (b) odd & of the form 3n
(c) odd & of the form (3n  1) (d) odd & of the form (3n + 1)

Q.14 (2n + 1) (2n + 3) (2n + 5) ....... (4n  1) is equal to :


(4 n) ! (4 n) ! n !
(a) n
(b) n
2 . (2 n) ! (2 n) ! 2 . (2 n) ! (2 n) !
(4 n) ! n ! (4n) ! n !
(c) (d)
(2 n) ! (2 n) ! 2 n ! (2n) !

n3
1  195  P3
Q.15 The number of positive terms in the sequence xn = n   n 1
(n  N) is :
4  Pn  Pn 1
(a) 2 (b) 3 (c) 4 (d) 5

Q.16 If (C0 + C1) (C1 + C2) (C2 + C3) ...... (Cn1 + Cn) = m . C1C2C3 .... Cn1 , then m =
n 1 n 1
 n 1 (n  1)n  n  1 n n 1
(a) (b) (c) (d)
 n 1 ! n! n!  n  1 !

1  1  4 x  1  7 1  4 x  1  7 
Q.17 The expression      is a polynomial in x of
4x 1   2 
 
 2  

degree
(a) 7 (b) 5
(c) 4 (d) 3

Nothing is impossible, the word itself says ‘I’m possible’!


20 BINOMIAL THEOREM AND PERMUTATION & COMBINATION

Q.18 The sum of the series (1² + 1).1! + (2² + 1).2! + (3² + 1). 3! + ..... + (n² + 1). n! is :
(a) (n + 1). (n+2)! (b) n.(n+1)! (c) (n + 1). (n+1)! (d) none of these
n
n
 a  C3
Q.19 If the second term of the expansion  a1/13   is 14a5/2 then the value of
n
is :
 a 1  C2
(a) 4 (b) 3 (c) 12 (d) 6

Q.20 The numerically greatest terms in the expansion of (2x + 5y)34 when x = 3 & y = 2 is
(a) T21 (b) T22 (c) T23 (d) T24

Q.21 The value of 4 (nC1 + 4 . nC2 + 42 . nC3 + ...... + 4n  1) is :


(a) 0 (b) 5n + 1 (c) 5n (d) 5n  1

Q.22 The sum of the coefficients of all the even powers of x in the expansion of
(2x2  3x + 1)11 is :
(a) 2 . 610 (b) 3 . 610 (c) 611 (d) none

Q.23 The positive value of a so that the coefficient of x5 is equal to that of x15 in the
10
 a 
expansion of  x 2  3  is
 x 
1 1
(a) (b) (c) 1 (d) 2 3
2 3 3

Q.24 Set of value of r for which, 18C + 2 . 18Cr  1 + 18Cr  20C13 contains :
r2

(a) 4 elements (b) 5 elements (c) 7 elements (d) 10 elements


10
 x 1 x 1 
Q.25 In the expansion of  2/3 1/3
 1/2 
, the term which does not contain x is :
x  x 1 x  x 
(a) 10C0 (b) 10C7 (c) 10C4 (d) none

Q.26 Co-efficient of t in the expansion of,


( + p)m  1 + ( + p)m  2 ( + q) + ( + p)m  3 ( + q)2 + ...... ( + q)m  1
where  q and p  q is :

(a)
m

Ct pt  q t  (b)
m

Ct pm  t  qm  t 
pq p q

(c)
m

Ct pt  qt  (d)
m

Ct pm  t  qm t 
p q pq

CatalyseR Eduventures (India) Pvt. Ltd.


BINOMIAL THEOREM AND PERMUTATION & COMBINATION 21

18
 1 
Q.27 The term independent of ' x ' in the expansion of  9 x   , x > 0 , is  times
 3 x
the corresponding binomial co-efficient . Then '  ' is :
1 1
(a) 3 (b) (c)  (d) 1
3 3

Q.28 (1 + x) (1 + x + x2) (1 + x + x2 + x3) ...... (1 + x + x2 + ...... + x100) when written in the


ascending power of x then the highest exponent of x is ______ .
(a) 4950 (b) 5050 (c) 5150 (d) none

Q.29 The expression [x + (x31)1/2]5 + [x  (x31)1/2]5 is a polynomial of degree :


(a) 5 (b) 6 (c) 7 (d) 8
n
Q.30 
Let 5  2 6  = p + f where n  N and p  N and 0 < f < 1 then the value of,

f2  f + pf  p is :
(a) a natural number (b) a negative integer
(c) a prime number (d) are irrational number

Q.31 Given (1 – 2x + 5x2 – 10x3) (1 + x)n = 1 + a1x + a2x2 + .... and that a12 = 2a2 then the
value of n is
(a) 6 (b) 2 (c) 5 (d) 3

Q.32 If (1 + x – 3x2)2145 = a0 + a1x + a2x2 + ......... then a0 – a1 + a2 – a3 + ..... ends with


(a) 1 (b) 3 (c) 7 (d) 9

Q.33 The remainder, if 1 + 2 + 22 + 23 + ......+21999 is divided by 5 is


(a) 0 (b) 1 (c) 2 (d) 3
n
 q p7 
Q.34 In the expansion of   10 3  , there is a term similar to pq, then that term is equal to
 p q 

(a) 210 pq (b) 252 pq
(c) 120 pq (d) 45 pq
12
 1 1
 
34  43 
Q.35 Sum of all the rational terms is the expansion of   , is
(a) 27 (b) 256
(c) 283 (d) none

Nothing is impossible, the word itself says ‘I’m possible’!


22 BINOMIAL THEOREM AND PERMUTATION & COMBINATION

Q.36 The number of values of ' r ' satisfying the equation ,


39
C3r  1  39Cr2 = 39
Cr 2  1  39
C3 r is :

(a) 1 (b) 2 (c) 3 (d) 4


100
Q.37 Number of rational terms in the expansion of  2  43  is :

(a) 25 (b) 26 (c) 27 (d) 28

 54 k  x k  8
4
Q.38 The largest real value for x such that      is
k 0  (4  k )!  k !  3
(a) 2 2  5 (b) 2 2  5 (c) 2 2  5 (d) 2 2  5

1
Q.39 Greatest term in the binomial expansion of (a + 2x)9 when a = 1 & x = is :
3
(a) 3rd & 4th (b) 4th & 5th (c) only 4th (d) only 5th
Q.40 The coefficient of the middle term in the binomial expansion in powers of x of (1 + x)4
and of (1 – x)6 is the same if  equals
5 10 3 3
(a) – (b) (c) – (d)
3 3 10 5
Q.41 Last two digits of 21 to the (100)th power, is
(a) 00 (b) 01 (c) 11 (d) 21
Choose the correct alternatives. (More than one are correct)
11
Q.42 In the expansion of  x 3  3.2 2  :
 log x3

 
(a) there appears a term with the power x2
(b) there does not appear a term with the power x2
(c) there appears a term with the power x 3
1
(d) the ratio of the co-efficient of x3 to that of x 3 is
3
5
Q.43 If it is known that the third term of the binomial expansion  x  x log 10 x
 is 106 then x
is equal to :
(a) 10 (b) 10  5/2 (c) 100 (d) 5
Q.44 Let (1 + x2)2 (1 + x)n = A0 + A1 x + A2 x2 + ...... If A0, A1, A2 are in A.P. then the value
of n is :
(a) 2 (b) 3 (c) 5 (d) 7
Q.45 The number 101100  1 is divisible by
(a) 100 (b) 1000 (c) 10000 (d) 100000

CatalyseR Eduventures (India) Pvt. Ltd.


BINOMIAL THEOREM AND PERMUTATION & COMBINATION 23

ANSWER KEY
1 2 3 4 5 6 7 8 9 10 11 12 13 14 15
C A C A A B C B C A D D A B C
16 17 18 19 20 21 22 23 24 25 26 27 28 29 30
B D B A B D B A C C B D B C B
31 32 33 34 35 36 37 38 39 40 41 42 43 44 45
A B A B C B B A B C B BCD AB AB ABC

1.4. ADVANCE SUBJECTIVE EXERCISES:

ADVANCE SUBJECTIVE EXERCISE - 1


11 11
 1   1 
Q.1 Find the coefficients : (i) x7 in  a x2   (ii) x7 in  ax  2 
 bx  bx 
(iii) Find the relation between a & b , so that these coefficients are equal.

Q.2 If the coefficients of the rth, (r + 1)th & (r + 2)th terms in the expansion of (1 + x)14
are in AP, find r.

Q.3 Find the term independent of x in the expansion of


10 8
 x 3 1 
(a)   2 (b)  x1/3  x 1/5 
 3 2x  2 

Q.4 If the coefficients of 2nd , 3rd & 4th terms in the expansion of (1 + x)2n are in AP,
show that
2n²  9n + 7 = 0.

Q.5 Given that (1 + x + x²)n = a0 + a1x + a2x² + .... + a2nx2n , find the values of :
(i) a0 + a1 + a2 + ..... + a2n
(ii) a0  a1 + a2  a3 ..... + a2n
(iii) a02 a12 + a22  a32 + ..... + a2n2

Q.6 If a, b, c & d are the coefficients of any four consecutive terms in the expansion of
a c 2b
(1 + x)n, n  N, prove that   .
ab cd bc

Nothing is impossible, the word itself says ‘I’m possible’!


24 BINOMIAL THEOREM AND PERMUTATION & COMBINATION

Q.7 Find the value of x for which the fourth term in the expansion,
8
 52 log5 4 x  44 1 
5  3 2 x 1  7  is 336.
 5log5 

Q.8 Prove that : n1C + n2Cr + n3Cr + .... + rCr = nCr+1.


r

Q.9 (a) Which is larger : (9950 + 10050) or (101)50.


2n–2C
4n
(b) Show that n–2 + 2.2n–2Cn–1 + 2n–2Cn > , n N , n  2
n 1
11
 7
Q.10 In the expansion of  1  x   find the term not containing x.
 x

Q.11 Show that coefficient of x5 in the expansion of (1 + x²)5 . (1 + x)4 is 60.

Q.12 Find the coefficient of x4 in the expansion of :


(i) (1 + x + x2 + x3)11 (ii) (2  x + 3x2)6

Q.13 Find numerically the greatest term in the expansion of :


3
(i) (2 + 3x)9 when x =
2
1
(ii) (3  5x)15 when x =
5
2 n
q1  q 1   q 1 
Q.14 Given sn= 1 + q + q² + ..... + qn & Sn = 1 + +  + .... +   , q  1,
2  2   2 
prove that n+1C + n+1C2.s1 + n+1C3.s2 +....+ n+1Cn+1.sn = 2n . Sn .
1

Q.15 Prove that the ratio of the coefficient of x10 in (1  x²)10 & the term independent
10
 2
of x in  x   is 1 : 32 .
 x

n4
Q.16 Let (1+x²)² . (1+x)n =  aK . x K . If a1 , a2 & a3 are in AP, find n.
K 0

(1  xn )(1  xn 1 )(1  x n2 )..................(1  x nr 1 )


Q.17 If nJr = 2 3 r , prove that nJn – r = nJr.
(1  x)(1  x )(1  x )..................(1  x )
n
Q.18 Prove that  n C K sin Kx .cos( n  K ) x  2 n 1 sin nx .
K 0

CatalyseR Eduventures (India) Pvt. Ltd.


BINOMIAL THEOREM AND PERMUTATION & COMBINATION 25

Q.19 The expressions 1 + x, 1+x + x2, 1 + x + x2 + x3,............. 1 + x + x2 +........... + xn are


multiplied together and the terms of the product thus obtained are arranged in increasing
powers of x in the form of a0 + a1x + a2x2 +................., then,
(a) how many terms are there in the product.
(b) show that the coefficients of the terms in the product, equidistant from the beginning
and end are equal.
(n  1)!
(c) show that the sum of the odd coefficients = the sum of the even coefficients =
2

Q.20 Find the coeff. of (a) x6 in the expansion of (ax² + bx + c)9 .


(b) x2 y3 z4 in the expansion of (ax  by + cz)9 .
(c) a2 b3 c4 d in the expansion of (a – b – c + d)10.
2n 2n
r
Q.21 If a r ( x  2)  b r ( x  3) r & ak = 1 for all k  n, then show that bn = 2n+1Cn+1.
r 0 r 0

Q.22 Find the coefficient of xr in the expression of :


(x + 3)n1 + (x + 3)n2 (x + 2) + (x + 3)n3 (x + 2)2 + ..... + (x + 2)n1
n
 x 2
Q.23 (a) Find the index n of the binomial    if the 9th term of the expansion
5 5
has numerically the greatest coefficient (n  N) .
(b) For which positive values of x is the fourth term in the expansion of (5 + 3x)10 is
the greatest.

(72)!
Q.24 Prove that 2
 1 is divisible by 73.
 36!
Q.25 Find the number of divisors of the number
N = 2000C1 + 2 · 2000C2 + 3 · 2000C3 + ...... + 2000 · 2000C2000.

Q.26 Show that the integral part in each of the following is odd. n  N
n n

(a) 5  2 6  
(b) 8  3 7 
n
(c) 6  35 

Q.27 Show that the integral part in each of the following is even. n N
2 n 1 2 n 1

(a) 3 3  5  
(b) 5 5  11 

Nothing is impossible, the word itself says ‘I’m possible’!


26 BINOMIAL THEOREM AND PERMUTATION & COMBINATION
n
 
Q.28 If 7  4 3 = p+ where n & p are positive integers and  is a proper fraction show that

(1  ) (p + ) = 1.
2001
1 
Q.29 Find the sum of the roots (real or complex) of the equation x2001 +   x  = 0.
2 
n
Q.30 Let I denotes the integral part & F the proper fractional part of 3  5   where n  N

and if  denotes the rational part and  the irrational part of the same, show that
1 1
 = (I + 1) and  = (I + 2 F  1).
2 2

ANSWER KEY

11C a6 11C a5
Q.1 (i) 5 (ii) 6 (iii) ab = 1
b5 b6
5
Q.2 r = 5 or 9 Q.3 (a) (b) T6 =7
12
Q.5 (i) 3n (ii) 1, (iii) an Q.7 x = 0 or 1
Q.9 (a) 10150 (Prove that 10150  9950 = 10050 + some +ive qty)
5
Q.10 1 +  11C
2k . 2kCk 7k
k 1

7.313
Q.12 (i) 990 (ii) 3660 Q.13 (i) T7 = (ii) 455 x 312
2
Q.16 n = 2 or 3 or 4
n2  n  2
Q.19 (a)
2
Q.20 (a) 84b6c3 + 630ab4c4 + 756a2b2c5 + 84a3c6; (b) 1260 . a2b3c4 ; (c) 12600
nC (3 nr 
5 20
Q.22 r 2nr) Q.23 (a) n = 12 (b) <x<
8 21
Q.25 8016 Q.29 500

CatalyseR Eduventures (India) Pvt. Ltd.


BINOMIAL THEOREM AND PERMUTATION & COMBINATION 27

ADVANCE SUBJECTIVE EXERCISE - 2

(On combinatorial coefficients)

If C0 , C1 , C2 , ..... , Cn are the combinatorial coefficients in the expansion of


(1 + x)n, n  N , then prove the following :

Q.1 C0² + C1² + C2² +.....+ Cn² =


 2 n !
n! n !

Q.2 C0 C1 + C1 C2 + C2 C3 +....+Cn1 Cn =
 2 n !
(n  1)! (n 1)!

Q.3 C1 + 2C2 + 3C3 +.....+ n . Cn = n . 2n1

Q.4 C0 + 2C1 + 3C2 +.....+ (n+1)Cn = (n+2)2n1

Q.5 C0 + 3C1 + 5C2 +.....+ (2n+1)Cn = (n+1) 2n

C0 . C1 . C2 .... Cn 1 (n  1) n
Q.6 (C0+C1)(C1+C2)(C2+C3) ..... (Cn1+Cn) =
n!

Q.7 If Pn denotes the product of all the coefficients in the expansion of (1 + x) n, n  N, show
Pn1 (n  1)n
that, 
Pn n!

C1 2 C2 3 C3 n . Cn n (n  1)
Q.8    .......  
C0 C1 C2 Cn 1 2

22 . C1 23 . C2 24 . C3 2 n 1 . Cn 3n 1 1
Q.9 2 . Co +    ...... 
2 3 4 n 1 n 1

2 n!
Q.10 CoCr + C1Cr+1 + C2Cr+2 + .... + Cnr Cn =
(n  r )! ( n  r )!

C1 C2 Cn 1
Q.11 Co    ......  (  1) n 
2 3 n 1 n 1

(n  1) (2 n)!
Q.12 1 . Co² + 3 . C1² + 5 . C2² + ..... + (n+1) Cn² =
n! n!

Nothing is impossible, the word itself says ‘I’m possible’!


28 BINOMIAL THEOREM AND PERMUTATION & COMBINATION

Q.13 If a0 , a1 , a2 , ..... be the coefficients in the expansion of (1 + x + x²)n in ascending


powers of x , then prove that :
(i) a0 a1  a1 a2 + a2 a3  .... = 0 .
(ii) a0a2  a1a3 + a2a4  ..... + a2n  2 a2n = an + 1 or an–1.

(iii) E1 = E2 = E3 = 3n1 ; where E1= a0 + a3 + a6 + ..... ; E2 = a1 + a4 + a7 + ..... &


E3 = a2 + a5 + a8 + .....

Q.14 If (1+x)n = C0 + C1x + C2x² + .... + Cn xn , then show that the sum of the products of
  Ci C j
theCi’s taken two at a time , represented by is
0i jn
2 n!
equal to 22n1  .
2( n !) 2

Q.15 If (1 + x + x² + ... + xp)n = a0 + a1x + a2x²+...+anp. xnp , then find the value of :
a1 + 2a2 + 3a3 + .... + np . anp

Q.16 1². C0 + 2². C1 + 3². C2 + 4². C3 + .... + (n+1)² Cn = 2n2 (n+1) (n+4) .

C1 C C Cn (1  x)n 1 1
Q.17 C0 + x  2 x 2  3 x 3  .......  . xn =
2 3 4 n 1 (n  1) x

C0 C2 C4 2n
Q.18    ........ 
1 3 5 n 1

C0 C1 C2 C3 Cn 1  n .2n 1
Q.19     ........  
2 3 4 5 n  2 (n  1) (n  2)

(4 n  1)!
Q.20 Prove that , (2nC1)²+ 2 . (2nC2)² + 3 . (2nC3)² + ... + 2n . (2nC2n)² = 2
(2 n  1) !

ANSWER KEY
np
Q15 Differentiate the given expn. & put x = 1 to get the result (p+1)n
2
1
Q.18 Consider [(1+x)n + (1x)n] = C0 + C2x² + C4x4 + ..... Integrate between 0 & 1.
2
Q.19 Multiply both sides by x the expn. (1+x)n . Integrate both sides between 0 & 1.

CatalyseR Eduventures (India) Pvt. Ltd.


BINOMIAL THEOREM AND PERMUTATION & COMBINATION 29
PERMUTATIONS & COMBINATIONS 2
2.1. PERMUTATIONS & COMBINATIONS THEORY

2.1.1. The Factorial

Factorial notation: Let n be a positive integer. Then, the continued product of first n
natural numbers is called factorial n, to be denoted by n ! or n . Also, we define 0 ! = 1.
when n is negative or a fraction, n ! is not defined.
Thus, n ! = n (n – 1) (n – 2) ......3.2.1.
Deduction: n ! = n(n – 1) (n – 2) (n – 3) ......3.2.1
= n[(n  1)(n  2)(n  3)......3.2.1] = n [(n  1) ! ]

2.1.2. Exponent of Prime p in n !

Let p be a prime number and n be a positive integer.


Let E p (n) denotes the exponent of the prime p in the positive integer n. Then,
n  n   n   n 
E p (n ! )      2    3   ..... S 
 p  p   p  p 
where S is the largest natural number. Such that p S  n  p S  1 .
2.1.3. Fundamental Principles of Counting

(1) Addition principle : Suppose that A and B are two disjoint events (mutually
exclusive); that is, they never occur together. Further suppose that A occurs in m
ways and B in n ways. Then A or B can occur in m + n ways. This rule can also
be applied to more than two mutually exclusive events.
(2) Multiplication principle : Suppose that an event X can be decomposed into two stages
A and B. Let stage A occur in m ways and suppose that these stages are
unrelated, in the sense that stage B occurs in n ways regardless of the outcome
of stage A. Then event X occur in mn ways. This rule is applicable even if event X
can be decomposed in more than two stages.

Note :  The above principle can be extended for any finite number of operations and may be
stated as under :

If one operation can be performed independently in m different ways and if second


operation can be performed independently in n different ways and a third operation can
be performed independently in p different ways and so on, then the total number of
ways in which all the operations can be performed in the stated order
is (m × n × p × .....)

Nothing is impossible, the word itself says ‘I’m possible’!


30 BINOMIAL THEOREM AND PERMUTATION & COMBINATION

2.1.4. Definition of Permutation

The ways of arranging or selecting a smaller or an equal number of persons or objects at


a time from a given group of persons or objects with due regard being paid to the order of
arrangement or selection are called the (different) permutations.
For example : Three different things a, b and c are given, then different arrangements
which can be made by taking two things from three given things are ab, ac, bc, ba, ca, cb.
Therefore the number of permutations will be 6.

2.1.5. Number of Permutations without Repetition

(1) Arranging n objects, taken r at a time equivalent to filling r places from n things
1 2 3 4 r
n (n–1)(n – 2 )(n – 3) n – (r–1)
r-places :
Number of choices :The number of ways of arranging = The number of ways of filling r
places.= n(n  1)(n  2).......(n  r  1) =
n(n  1)(n  2).....(n  r  1)((n  r)! ) n!
  n Pr
(n  r )! (n  r )!

(2) The number of arrangements of n different objects taken all at a time = n Pn  n!

n n!
Note :  P0   1; n Pr  n . n 1 Pr 1
n!
1
 0 !  1;  0 or (r )!   (r  N )
(r )!

2.1.6. Number of Permutations with Repetition

(1) The number of permutations (arrangements) of n different objects, taken r at a time,


when each object may occur once, twice, thrice,........upto r times in any arrangement
= The number of ways of filling r places where each place can be filled by any one of
n objects.

1 2 3 4 r
n n n n n
r – places :
Number of choices :
The number of permutations = The number of ways of filling r places = (n)r

(2) The number of arrangements that can be formed using n objects out of which p are
identical (and of one kind) q are identical (and of another kind), r are identical (and of
n!
another kind) and the rest are distinct is .
p! q! r!

CatalyseR Eduventures (India) Pvt. Ltd.


BINOMIAL THEOREM AND PERMUTATION & COMBINATION 31

2.1.7. Conditional Permutations

(1) Number of permutations of n dissimilar things taken r at a time when p particular


n p
things always occur = Cr  p r!
(2) Number of permutations of n dissimilar things taken r at a time when p particular things
never occur = n  p C r r !
(3) The total number of permutations of n different things taken not more than r at a time,
n(nr  1)
when each thing may be repeated any number of times, is .
n1
(4) Number of permutations of n different things, taken all at a time, when m specified
things always come together is m !  (n  m  1) !
(5) Number of permutations of n different things, taken all at a time, when m specified
things never come together is n !m !  (n  m  1)!
(6) Let there be n objects, of which m objects are alike of one kind, and the remaining
(n  m) objects are alike of another kind. Then, the total number of mutually
n!
distinguishable permutations that can be formed from these objects is .
(m ! )  (n  m) !

Note :  The above theorem can be extended further i.e., if there are n objects, of which
p1 are alike of one kind; p 2 are alike of another kind; p 3 are alike of 3rd
kind;......: pr are alike of rth kind such that p1  p 2  ......  p r  n ; then the
n!
number of permutations of these n objects is .
( p1 ! )  ( p 2 ! )  ......  ( p r ! )

IMPORTANT TIPS
 Gap method : Suppose 5 males A, B, C, D, E are arranged in a row
as × A × B × C × D × E ×. There will be six gaps between these five. Four in between
and two at either end. Now if three females P, Q,R are to be arranged so that no two are
together we shall use gap method i.e., arrange them in between these 6 gaps. Hence the
6
answer will be P3 .

 Together : Suppose we have to arrange 5 persons in a row which can be done in


5 ! = 120 ways. But if two particular persons are to be together always, then we tie these
two particular persons with a string. Thus we have 5 – 2 + 1 (1 corresponding to these
two together) = 3 +1 = 4 units, which can be arranged in 4! ways. Now we loosen the
string and these two particular can be arranged in 2 ! ways. Thus total arrangements
= 24 × 2 = 48. Never together = Total – Together = 120 – 48 = 72.

Nothing is impossible, the word itself says ‘I’m possible’!


32 BINOMIAL THEOREM AND PERMUTATION & COMBINATION

2.1.8. Circular Permutations

In circular permutations, what really matters is the position of an object relative to the
others.
Thus, in circular permutations, we fix the position of the one of the objects and then
arrange the other objects in all possible ways.
There are two types of circular permutations :
(i) The circular permutations in which clockwise and the anticlockwise arrangements
give rise to different permutations, e.g. Seating arrangements of persons round a
table.
(ii) The circular permutations in which clockwise and
A A
the anticlockwise arrangements give rise to same
D B B D
permutations, e.g. arranging some beads to form a
C C
necklace.

Look at the circular permutations, given below :


Suppose A, B, C, D are the four beads forming a necklace. They have been arranged in
clockwise and anticlockwise directions in the first and second arrangements respectively.
Now, if the necklace in the first arrangement be given a turn, from clockwise to
anticlockwise, we obtain the second arrangement. Thus, there is no difference between
the above two arrangements.

(1) Difference between clockwise and anticlockwise arrangement : If anticlockwise


and clockwise order of arrangement are not distinct e.g., arrangement of beads in
a necklace, arrangement of flowers in garland etc. then the number of circular
(n  1) !
permutations of n distinct items is
2
(2) Theorem on circular permutations
Theorem 1 : The number of circular permutations of n different objects is (n  1) !
Theorem 2 : The number of ways in which n persons can be seated round a table is
(n  1)!
Theorem 3 : The number of ways in which n different beads can be arranged to form
1
a necklace, is (n  1)! .
2
Note :  When the positions are numbered, circular arrangement is treated as a linear
arrangement.
 In a linear arrangement, it does not make difference whether the positions are
numbered or not.

CatalyseR Eduventures (India) Pvt. Ltd.


BINOMIAL THEOREM AND PERMUTATION & COMBINATION 33

2.1.9. Definition Combinations


Each of the different groups or selections which can be formed by taking some or all of a
number of objects, irrespective of their arrangements, is called a combination.
Suppose we want to select two out of three persons A, B and C.
We may choose AB or BC or AC.
Clearly, AB and BA represent the same selection or group but they give rise to different
arrangements.
Clearly, in a group or selection, the order in which the objects are arranged is immaterial.
Notation: The number of all combinations of n things, taken r at a time is denoted by
 n
C (n, r) or n Cr or   .
r

(1) Difference between a permutation and combination :


(i) In a combination only selection is made whereas in a permutation not only a
selection is made but also an arrangement in a definite order is considered.
(ii) In a combination, the ordering of the selected objects is immaterial whereas in a
permutation, the ordering is essential. For example A, B and B, A are same as
combination but different as permutations.
(iii) Practically to find the permutation of n different items, taken r at a time, we first
select r items from n items and then arrange them. So usually the number of
permutations exceeds the number of combinations.
(iv) Each combination corresponds to many permutations. For example, the six
permutations ABC, ACB, BCA, BAC, CBA and CAB correspond to the same
combination ABC.

Note :  Generally we use the word ‘arrangements’ for permutations and word “selection”
for combinations.

2.1.10. Number of Combinations without Repetition

The number of combinations (selections or groups) that can be formed from n different
n!
objects taken r(0  r  n) at a time is n Cr 
r ! (n  r )!
Let the total number of selections (or groups) = x. Each group contains r objects, which can be
arranged in r ! ways. Hence the number of arrangements of r objects = x  (r! ) . But the number
n
Pr n!
of arrangements = n Pr .  x  (r! )  n Pr x x  n Cr .
r! r ! (n  r ) !

Nothing is impossible, the word itself says ‘I’m possible’!


34 BINOMIAL THEOREM AND PERMUTATION & COMBINATION

IMPORTANT TIPS
 n
Cr is a natural number.  n
C0  nCn  1, n C1  n

 n
Cr n Cn  r  n
Cr  n Cr 1  n1 Cr

 n
C x  n C y  x  y or x  y  n  n . n1 Cr 1  (n  r  1)n Cr 1

 If n is even then the greatest value of n


Cr is n
Cn / 2 .
n n
Cn  1 Cn 1
 If n is odd then the greatest value of n
Cr is or .
2 2
n
n n 1 Cr n r 1
 n
Cr  . Cr 1  n

r Cr 1 r

 n
C0  nC1  nC2  .....  nCn  2n  n
C0  nC2 n C4  ...... nC1 nC3  nC5  .....  2n 1

 2n 1
C0  2n 1C1  2n 1C2  .....  2n 1Cn  22n

 n
Cn  n 1Cn n  2Cn  n  3 Cn  ....  2n 1Cn  2nCn 1

Note:  Number of combinations of n dissimilar things taken all at a time


n! 1
n
Cn    1 , ( 0 !  1) . 5 C3  22 C9 .
n ! (n  n) ! 0 !

2.1.11. Number of Combinations with Repetition and all Possible Selections

(1) The number of combinations of n distinct objects taken r at a time when any object
may be repeated any number of times.
= coefficient of x r in (1  x  x 2  .......  x r ) n = coefficient of x r in (1  x )  n  n  r 1 C r

(2) The total number of ways in which it is possible to form groups by taking some or all
of n things at a time is 2 n  1 .

(3) The total number of ways in which it is possible to make groups by taking some or all
out of n  (n1  n 2  ....) things, when n1 are alike of one kind, n2 are alike of second
kind, and so on is {(n1  1)(n2  1)......}  1 .

(4) The number of selections of r objects out of n identical objects is 1.

(5) Total number of selections of zero or more objects from n identical objects is n  1 .

(6) The number of selections taking at least one out of a1  a 2  a 3  ......  an + k objects,
where a 1 are alike (of one kind), a 2 are alike (of second kind) and so on...... a n are
alike (of nth kind) and k are distinct = [( a 1  1) (a 2  1) (a 3  1).......( a n  1)] 2 k  1 .

CatalyseR Eduventures (India) Pvt. Ltd.


BINOMIAL THEOREM AND PERMUTATION & COMBINATION 35

2.1.12. Conditional Combinations


(1) The number of ways in which r objects can be selected from n different objects if k
particular objects are
nk n k
(i) Always included = C r k (ii) Never included = Cr

(2) The number of combinations of n objects, of which p are identical, taken r at a time is
= n  p C r  n  p C r  1  n  p C r  2  .......  n  p C 0 if r  p
n p
and = Cr  n p Cr 1  n p Cr 2  .......  n p Cr  p if r  p

2.1.13. Division into Groups

Case I :
(1) The number of ways in which n different things can be arranged into r
n  r 1 n 1
different groups is Pn or n ! C r 1 according as blank group are or are
not admissible.
(2) The number of ways in which n different things can be distributed into r different
group is r n  r C 1 (r  1) n  r C 2 (r  2) n  .........  (  1) n 1 n C r 1 or Coefficient of x n is n !
(e x  1) r Here blank groups are not allowed.

(3) Number of ways in which m × n different objects can be distributed equally among n
persons (or numbered groups) = (number of ways of dividing into groups) × (number
(mn) ! n ! (mn)!
of groups) ! =  .
(m ! )n n! (m! )n

Case II :

(1) The number of ways in which (m  n) different things can be divided into two groups
mn (m  n)!
which contain m and n things respectively is, C m .n C n  ,m  n .
m ! n!

Corollary: If m  n , then the groups are equal size. Division of these groups can be given by
two types.
Type I : If order of group is not important : The number of ways in which 2n different
(2n)!
things can be divided equally into two groups is
2! (n! ) 2
Type II : If order of group is important : The number of ways in which 2n different
(2n)! 2n!
things can be divided equally into two distinct groups is 2
 2! 
2! (n! ) (n! ) 2

Nothing is impossible, the word itself says ‘I’m possible’!


36 BINOMIAL THEOREM AND PERMUTATION & COMBINATION

(2) The number of ways in which (m + n + p) different things can be divided into three
groups which contain m, n and p things respectively is
mn p (m  n  p) !
C m .n  p C n . p C p  ,m  n  p
m!n! p !

Corollary: If m  n  p , then the groups are equal size. Division of these groups can be given
by two types.
Type I : If order of group is not important : The number of ways in which 3p different
(3 p)!
things can be divided equally into three groups is
3! (p! )3
Type II : If order of group is important : The number of ways in which 3p different
(3 p)! (3 p)!
things can be divided equally into three distinct groups is 3
t 3! 
3! ( p! ) ( p! )3

Note :  If order of group is not important : The number of ways in which mn different
mn!
things can be divided equally into m groups is
(n! )m m!
 If order of group is important: The number of ways in which mn different things
(mn)! (mn)!
can be divided equally into m distinct groups is m
 m!  .
(n! ) m! (n! )m

2.1.14. Derangement

Any change in the given order of the things is called a derangement.


If n things form an arrangement in a row, the number of ways in which they can be
deranged so that no one of them occupies its original place is
 1 1 1 1
n ! 1     ......  (1)n .  .
 1! 2! 3! n! 

2.1.15. Some Important Results for Geometrical Problems

(1) Number of total different straight lines formed by joining the n points on a plane of which
m (< n) are collinear is n C 2  m C 2  1 .

(2) Number of total triangles formed by joining the n points on a plane of which m (< n) are
collinear is n C 3  m C 3 .

(3) Number of diagonals in a polygon of n sides is n C 2  n .

(4) If m parallel lines in a plane are intersected by a family of other n parallel lines. Then
m mn(m  1)(n  1)
total number of parallelograms so formed is C 2  n C 2 i.e
4

CatalyseR Eduventures (India) Pvt. Ltd.


BINOMIAL THEOREM AND PERMUTATION & COMBINATION 37

(5) Given n points on the circumference of a circle, then


(i) Number of straight lines = n C 2 (ii) Number of triangles = n C 3
(iii) Number of quadrilaterals = n C 4 .
(6) If n straight lines are drawn in the plane such that no two lines are parallel and no three lines are
concurrent. Then the number of part into which these lines divide the plane is = 1  n .
n
3
(7) Number of rectangles of any size in a square of nn is r
r 1
and number of squares of

n
2
any size is r
r 1
.

np
(8) In a rectangle of n  p (n  p) number of rectangles of any size is (n  1)( p  1) and
4
n
number of squares of any size is  (n  1  r)(p  1  r) .
r 1

2.1.16. Multinomial Theorem


Let x1 , x 2 , ......., x m be integers. Then number of solutions to the equation
x1  x 2  ......  x m  n .....(i)
Subject to the condition a1  x 1  b1 , a 2  x 2  b 2 ,......., am  x m  bm .....(ii)
is equal to the coefficient of x n in
( x a1  x a1  1  ......  x b1 ) ( x a 2  x a 2 1  .....  x b 2 )......( x am  x a m  1  .....  x bm ) ......(iii)
This is because the number of ways, in which sum of m integers in (i) equals n, is the
same as the number of times x n comes in (iii).
(1) Use of solution of linear equation and coefficient of a power in expansions to find
the number of ways of distribution :
(i) The number of integral solutions of x1  x 2  x 3  ......  x r  n where
x1  0, x 2  0, ......x r  0 is the same as the number of ways to distribute n
identical things among r persons.
This is also equal to the coefficient of x n in the expansion of
r
0 1 2 3 r n  1  n
( x  x  x  x  ......) = coefficient of x in   = coefficient of x in
1 x 
n
(1  x ) r = coefficient of x in

 r(r  1) 2 r(r  1)(r  2)......(r  n  1) n


1  rx  x  ......  x  .......
 2! n!
r(r  1)(r  2)....(r  n  1) (r  n  1)! n r 1
   Cr 1
n! n!(r  1)!

Nothing is impossible, the word itself says ‘I’m possible’!


38 BINOMIAL THEOREM AND PERMUTATION & COMBINATION

(ii) The number of integral solutions of x1  x 2  x 3  .....  x r  n where


x1  1, x 2  1,.......x r  1 is same as the number of ways to distribute n identical
things among r persons each getting at least 1. This also equal to the coefficient of
r
 x 
x n in the expansion of = coefficient of x n in 
1 2 3 4 r
n (n–1) (n – 2 )(n – 3) n – (r–1)  =
1 x 
coefficient of x n in x r (1  x ) r = coefficient of x n in
 r (r  1) 2 r (r  1)(r  2).....(r  n  1) n 
x r 1  rx  x  .....  x  .....
 2! n! 
1 2 3 4 r

= coefficient of n n n n n in
 r(r  1) 2 r(r  1)(r  2).....(r  n  1) n 
1  rx  x  .....  x  .....
 2! n! 
r (r  1) (r  2)......( r  n  r  1) r(r  1) (r  2).....(n  1)
= =
(n  r ) ! (n  r )!
(n  1) !
=  n  1 Cr  1 .
(n  r )! (r  1)!

2.1.17. Number of divisors

Let N  p 1 . p 2 . p 3 ...... p k , where p1 , p 2 , p 3 ,......p k are different primes and


1 2 3 k

 1 , 2 , 3 ,......,  k are natural numbers then :


(1) The total number of divisors of N including 1 and N is =
( 1  1) ( 2  1) ( 3  1)....( k  1)
(2) The total number of divisors of N excluding 1 and N is =
( 1  1) ( 2  1) ( 3  1).....( k  1)  2
(3) The total number of divisors of N excluding 1 or N is =
(1  1)( 2  1)( 3  1).....( k  1)  1
(4) The sum of these divisors is
 ( p10  p 12  p 32  ......  p1 1 ) ( p 20  p 12  p 22  ...  p 2 2 ).....( p k0  p 1k  p k2  ....  p k k )
(5) The number of ways in which N can be resolved as a product of two factors is
1
 2 (1  1)( 2  1)....( k  1), If N is not a perfect square
1
 [(1  1)( 2  1).....( k  1)  1], If N is a perfect square
2

(6) The number of ways in which a composite number N can be resolved into two factors
which are relatively prime (or co-prime) to each other is equal to 2 n1 where n is the
number of different factors in N.

CatalyseR Eduventures (India) Pvt. Ltd.


BINOMIAL THEOREM AND PERMUTATION & COMBINATION 39

IMPORTANT TIPS

 All the numbers whose last digit is an even number 0, 2, 4, 6 or 8 are divisible by 2.
 All the numbers sum of whose digits are divisible by 3,is divisible by 3 e.g. 534. Sum of
the digits is 12, which are divisible by 3, and hence 534 is also divisible by 3.
 All those numbers whose last two-digit number is divisible by 4 are divisible by 4 e.g.
7312, 8936, are such that 12, 36 are divisible by 4 and hence the given numbers are also
divisible by 4.
 All those numbers, which have either 0 or 5 as the last digit, are divisible by 5.
 All those numbers, which are divisible by 2 and 3 simultaneously, are divisible by 6. e.g.,
108, 756 etc.
 All those numbers whose last three-digit number is divisible by 8 are divisible by 8.
 All those numbers sum of whose digit is divisible by 9 are divisible by 9.
 All those numbers whose last two digits are divisible by 25 are divisible by 25 e.g.,
73125, 2400 etc.

Nothing is impossible, the word itself says ‘I’m possible’!


40 BINOMIAL THEOREM AND PERMUTATION & COMBINATION

2.2. SOLVED EXAMPLES:

1. In a chass tournament, all participants were to play one game with the other. Two
players fell ill after having played 3 games each. If total number of games played
in the tournament is equal to 84, then total number of participants in the beginning
was equal to:
(a) 10 (b) 15
(c) 12 (d) 14
Solution : (b) Let there were ‘n’ players in the beginning. Total number of games to be played
was played to nC2 and each player would have played (n – 1) games.
Thus
n
C2 – ((n – 1) + (n – 1) – 1) + 6 = 84
 n2 – 5n – 150 = 0  n = 15
2. Number of ways in which four letter of the word ‘DEGREE’ can be selected is
(a) 7 (b) 6
6!
(c) (d) none of these.
3!

Solution: (a) In DEGREE we have three E’s and D, G, R Four letters can be selected in
following ways
3
(i) Three alike, one different letter C1  3C3
(ii) Two alike, two different letter 3C2
(iii) all different letter 3C3
Total number of ways = 3C1 + 3C2 + 3C3 = 7.

3. A is a set containing n elements. A subset P of A is chosen. The set A is


reconstructed by replacing the elements of P. A subset Q of A is again chosen.
The number of ways of chosen P and Q so that P Q =  is
(a) 22n – 2nCn (b) 2n
(c) 2n – 1 (d) 3n

Solution: (d) Let A = { a1, a2, a3, . . . , an} . For ai  A, we have the following choices:
(i) ai  P and aiQ
(ii) ai  P and aiQ
(iii) ai P and aiQ
(iv) ai  P and aiQ
Out of these only (ii) , (iii) and (iv) imply a i  P  Q. Therefore, the number of
ways in which none of a1, a2, . . .an belong to P  Q is 3n .

CatalyseR Eduventures (India) Pvt. Ltd.


BINOMIAL THEOREM AND PERMUTATION & COMBINATION 41

4. If n objects are arranged in a row, then the number of ways of selecting three
objects so that no two of them are next to each other is
(a)  n  2  n  3 n  4  (b) n-2C3
6
(c) n-3C3 + n-3C2 (d) none of these.
Solution (a) , (b) , (c)
Let x0 be the number of objects to the left of the first object chosen, x1 the
number of objects between the first and the second, x2 the number of objects
between the second and the third and x3 the number of objects to the right of
the third object.
We have x0 , x3  0 , x1, x2  1 and x0 + x1 +x2 + x3 = n –3 . . . (1)
The number of solutions of (1) = coefficient of yn-3 in
(1+ y+ y2+...)(1+ y + y2+ ...)(y + y2+ y3 + ...)(y + y2+ y3+...)= coefficient of yn-3
in y2( 1+ y + y2 +y3+ . . .)4 = coefficient of yn-5 in (1- y)-4 =
coefficient of yn-5 in ( 1+ 4C1y + 5C2 y2 + 6C3y3 + . . .) = n-5 +3Cn-5 = n-2C3 =
n  2n  3 n  4  also n-3 C + n-3C = n-2C .
3 2 3
6

5. Five balls of different colours are to be placed in three boxes of different sizes.
Each box can hold all five balls. The number of ways in which we can place the
balls in the boxes ( order is not considered in the box) so that no box remains
empty is
(a) 150 (b) 300 (c) 200 (d) none of these
Solution: (a) One possible arrangement is 2 2 1

Three such arrangements are possible. Therefore, the number of ways is


( 5C2) ( 3C2) ( 1C1) (3) = 90
The other possible arrangements 1 1 2

Three such arrangements are possible. In this case, the number of ways is
( 5C1) ( 4C1) ( 3C3) (3) = 60
Hence, the total number of ways is 90 + 60 = 150.

6. Total number of 4 digit number that are greater than 3000, that can be formed
using the digits 1, 2, 3, 4, 5, 6 (no digit is being repeated in any number) is equal
to:
(a) 120 (b) 240 (c) 480 (d) 80
Solution : (b) Let the formed number is x1 x2 x3 x4
Clearly, x1 > 3.
Thus total number of such numbers = 4.5.4.3 = 240

Nothing is impossible, the word itself says ‘I’m possible’!


42 BINOMIAL THEOREM AND PERMUTATION & COMBINATION

7. The number of ways in which 6 men can be arranged in a row so that three
particular men are consecutive, is
(a) 4P4 (b) 4P4  3P3 (c) 6P6  3P3 (d) 3P3  3P3
Soution: (b) Considering three particular persons as a single group. Number of ways in which
these four can be arranged in a row is 4P4. Those three can arrange themselves in
3
P3 ways. So total number of ways = 4P4  3P3.

8. A convex polygon has 44 diagonals. The number of it’s sides is equal to:
(a) 9 (b) 10 (c) 11 (d) 12
Solution : (c) If number of sides is n, then
Total number of diagonals of a convex polygon = nC2 – n = 44 (given)
 n (n – 1) – 2n = 88  n2 – 3n – 88 = 0  (n –11) ( n + 8) = 0
 n = 11.



CatalyseR Eduventures (India) Pvt. Ltd.


BINOMIAL THEOREM AND PERMUTATION & COMBINATION 43

2.3. ADVANCE OBJECTIVE EXERCISES:

ADVANCE OBJECTIVE EXERCISE - 1


(Fundamental Principle of Counting)

Q.1 In how many ways can clean & clouded (overcast) days occur in a week
assuming that an entire day is either clean or clouded.
Q.2 Four visitors A, B, C & D arrive at a town which has 5 hotels. In how many ways
can they disperse themselves among 5 hotels, if 4 hotels are used to accommodate
them.
Q.3 If the letters of the word “VARUN” are written in all possible ways and then are
arranged as in a dictionary, then the rank of the word VARUN is :
(a) 98 (b) 99 (c) 100 (d) 101
Q.4 How many natural numbers are their from 1 to 1000 which have none of their digits
repeated.

Q.5 A man has 3 jackets, 10 shirts, and 5 pairs of slacks. If an outfit consists of a jacket, a
shirt, and a pair of slacks, how many different outfits can the man make?
Q.6 There are 6 roads between A & B and 4 roads between B & C.
(i) In how many ways can one drive from A to C by way of B?
(ii) In how many ways can one drive from A to C and back to A, passing through B
on both trips ?
(iii) In how many ways can one drive the circular trip described in (ii) without using
the same road more than once.
Q.7 (i) How many car number plates can be made if each plate contains 2 different letters
of English alphabet, followed by 3 different digits.
(ii) Solve the problem, if the first digit cannot be 0. (Do not simplify)
Q.8 (i) Find the number of four letter word that can be formed from the letters of the
word HISTORY. (each letter to be used at most once)
(ii) How many of them contain only consonants?
(iii) How many of them begin & end in a consonant?
(iv) How many of them begin with a vowel?
(v) How many contain the letters Y?
(vi) How many begin with T & end in a vowel?
(vii) How many begin with T & also contain S?
(viii) How many contain both vowels?

Nothing is impossible, the word itself says ‘I’m possible’!


44 BINOMIAL THEOREM AND PERMUTATION & COMBINATION

Q.9 If repetitions are not permitted


(i) How many 3 digit numbers can be formed from the six digits 2, 3, 5, 6, 7 & 9 ?
(ii) How many of these are less than 400 ?
(iii) How many are even ?
(iv) How many are odd ?
(v) How many are multiples of 5 ?
Q.10 In how many ways can 5 letters be mailed if there are 3 different mailboxes available if
each letter can be mailed in any mailbox.

Q.11 Every telephone number consists of 7 digits. How many telephone numbers are there
which do not include any other digits but 2 , 3 , 5 & 7 ?

Q.12 (a) In how many ways can four passengers be accommodate in three railway
carriages, if each carriage can accommodate any number of passengers.
(b) In how many ways four persons can be accommodated in 3 different chairs if each
person can occupy only one chair.

Q.13 How many of the arrangements of the letter of the word “LOGARITHM” begin with a
vowel and end with a consonant?

Q.14 Number of natural numbers between 100 and 1000 such that at least one of their digits is
7, is
(a) 225 (b) 243 (c) 252 (d) none

Q.15 How many four digit numbers are there which are divisible by 2 .

Q.16 In a telephone system four different letter P, R, S, T and the four digits 3, 5, 7, 8 are used.
Find the maximum number of “telephone numbers” the system can have if each consists
of a letter followed by a four-digit number in which the digit may be repeated.

Q.17 Find the number of 5 lettered palindromes which can be formed using the letters from
the English alphabets.

Q.18 Number of ways in which 7 different colours in a rainbow can be arranged if green is
always in the middle.

Q.19 Two cards are drawn one at a time & without replacement from a pack of 52 cards.
Determine the number of ways in which the two cards can be drawn in a definite order.

Q.20 Numbers of words which can be formed using all the letters of the word "AKSHI", if
each word begins with vowel or terminates in vowel .

CatalyseR Eduventures (India) Pvt. Ltd.


BINOMIAL THEOREM AND PERMUTATION & COMBINATION 45

Q.21 A letter lock consists of three rings each marked with 10 different letters. Find the
number of ways in which it is possible to make an unsuccessful attempts to open the lock.
Q.22 How many 10 digit numbers can be made with odd digits so that no two consecutive
digits are same.

Q.23 It is required to seat 5 men and 4 women in a row so that the women occupy the even
places. How many such arrangements are possible?

Q.24 If no two books are alike, in how many ways can 2 red, 3 green, and 4 blue books be
arranged on a shelf so that all the books of the same colour are together?

Q.25 How many natural numbers are there with the property that they can be expressed as the
sum of the cubes of two natural numbers in two different ways.



ANSWER KEY

Q.1 128 Q.2 120 Q.3 C Q.4 738 Q.5 150

Q.6 (i) 24 (ii) 576 (iii) 360 Q.7 (i) 468000 (ii) 421200

Q.8 (i) 840 (ii) 120 (iii) 400 (iv) 240 (v) 480 (vi) 40
(vii) 60 (viii) 240

Q.9 (i) 120 (ii) 40 (iii) 40 (iv) 80 (v) 20

Q.10 243 ways Q.11 47 Q.12 (a) 34 (b) 24 Q.13 90720

Q.14 C Q.15 4500 Q.16 1024 Q.17 263 Q.18 720 Q.19 2652

Q.20 84 Q.21 999 Q.22 5·49 Q.23 2880 Q.24 1728

Q.25 Infinitely many

Nothing is impossible, the word itself says ‘I’m possible’!


46 BINOMIAL THEOREM AND PERMUTATION & COMBINATION

ADVANCE OBJECTIVE EXERCISE – 2

Q.1 How many of the 900 three digit numbers have at least one even digit?
(a) 775 (b) 875 (c) 450 (d) 750

Q.2 The number of natual numbers from 1000 to 9999 (both inclusive) that do not have all 4
different digits is
(a) 4048 (b) 4464 (c) 4518 (d) 4536
OR
What can you say about the number of even numbers under the same constraints?

Q.3 The number of different seven digit numbers that can be written using only three digits
1, 2 & 3 under the condition that the digit 2 occurs exactly twice in each number is :
(a) 672 (b) 640 (c) 512 (d) none

Q.4 Out of seven consonants and four vowels, the number of words of six letters, formed by
taking four consonants and two vowels is (Assume that each ordered group of letter is a
word):
(a) 210 (b) 462 (c) 151200 (d) 332640

Q.5 All possible three digits even numbers which can be formed with the condition that if 5
is one of the digit, then 7 is the next digit is :
(a) 5 (b) 325 (c) 345 (d) 365

Q.6 For some natural N , the number of positive integral ' x ' satisfying the equation ,
1 ! + 2 ! + 3 ! + ...... + (x !) = (N)2 is :
(a) none (b) one (c) two (d) infinite

Q.7 The number of six digit numbers that can be formed from the digits 1, 2, 3, 4, 5, 6 & 7
so that digits do not repeat and the terminal digits are even is :
(a) 144 (b) 72 (c) 288 (d) 720
Q.8 A new flag is to be designed with six vertical strips using some or all of the colours
yellow, green, blue and red. Then, the number of ways this can be done such that no two
adjacent strips have the same colour is
(a) 12 × 81 (b) 16 × 192 (c) 20 × 125 (d) 24 × 216

Q.9 In how many ways can 5 colours be selected out of 8 different colours including red,
blue, and green
(a) if blue and green are always to be included,
(b) if red is always excluded,
(c) if red and blue are always included but green excluded?

CatalyseR Eduventures (India) Pvt. Ltd.


BINOMIAL THEOREM AND PERMUTATION & COMBINATION 47

Q.10 A 5 digit number divisible by 3 is to be formed using the numerals 0, 1, 2, 3, 4 & 5


without repetition. The total number of ways this can be done is :
(a) 3125 (b) 600 (c) 240 (d) 216

Q.11 Number of 9 digits numbers divisible by nine using the digits from 0 to 9 if each digit is
used atmost once is K . 8 ! , then K has the value equal to ______ .

Q.12 Number of natural numbers less than 1000 and divisible by 5 can be formed with the ten
digits, each digit not occuring more than once in each number is ______ .

ANSWER KEY
Q.1 A Q.2 B Q.3 A Q.4 C Q.5 D Q.6 C

Q.7 D Q.8 A Q.9 (a) 20, (b) 21, (c) 10 Q.10 D

Q.11 K = 17 Q.12 154

ADVANCE OBJECTIVE EXERCISE – 3

Q.1 Find the number of ways in which letters of the word VALEDICTORY be arranged so
that the vowels may never be separated.

Q.2 How many numbers between 400 and 1000 (both exclusive) can be made with the digits
2,3,4,5,6,0 if
(a) repetition of digits not allowed.
(b) repetition of digits is allowed.

Q.3 Number of odd integers between 1000 and 8000 which have none of their digits repeated,
is
(a) 1014 (b) 810
(c) 690 (d) 1736

Q.4 If 20Pr = 13× 20Pr–1 , then the value of r is ___________.

Q.5 The number of ways in which 5 different books can be distributed among 10 people if
each person can get at most one book is :
(a) 252 (b) 105
(c) 510 (d) 10C5.5!

Nothing is impossible, the word itself says ‘I’m possible’!


48 BINOMIAL THEOREM AND PERMUTATION & COMBINATION

Q.6 The product of all odd positive integers less than 10000, is
(10000 )! (10000)! (9999)! (10000 )!
(a) (b) (c) (d)
( 5000 !)2 25000 25000 5000
2 ·(5000 )!

Q.7 The 9 horizontal and 9 vertical lines on an 8 × 8 chessboard form 'r' rectangles and 's'
s
squares. The ratio in its lowest terms is
r
1 17 4
(a) (b) (c) (d) none
6 108 27

Q.8 There are 720 permutations of the digits 1, 2, 3, 4, 5, 6. Suppose these permutations are
arranged from smallest to largest numerical values, beginning from 1 2 3 4 5 6 and
ending with 6 5 4 3 2 1.
(a) What number falls on the 124th position?
(b) What is the position of the number 321546?

Q.9 A student has to answer 10 out of 13 questions in an examination . The number of ways
in which he can answer if he must answer atleast 3 of the first five questions is :
(a) 276 (b) 267 (c) 80 (d) 1200

Q.10 The number of three digit numbers having only two consecutive digits identical is
(a) 153 (b) 162 (c) 180 (d) 161

Q.11 Number of 3 digit numbers in which the digit at hundreath's place is greater than the
other two digit is
(a) 285 (b) 281 (c) 240 (d) 204

Q.12 Number of permutations of 1, 2, 3, 4, 5, 6, 7, 8 and 9 taken all at a time are such that the
digit
1 appearing somewhere to the left of 2
3 appearing to the left of 4 and
5 somewhere to the left of 6, is (e.g. 815723946 would be one such permutation)
(a) 9 · 7! (b) 8! (c) 5! · 4! (d) 8! · 4!

ANSWER KEY
Q.1 967680 Q.2 (a) 60 (b) 107 Q.3 D Q.4 r = 8 Q.5 D
th
Q.6 D Q.7 B Q.8 (a) 213564, (b) 267
Q.9 A Q.10 B Q.11 A Q.12 A

CatalyseR Eduventures (India) Pvt. Ltd.


BINOMIAL THEOREM AND PERMUTATION & COMBINATION 49

ADVANCE OBJECTIVE EXERCISE – 4

Q.1 A telegraph has x arms & each arm is capable of (x  1) distinct positions, including the
position of rest. The total number of signals that can be made is ______ .

Q.2 The interior angles of a regular polygon measure 150º each . The number of diagonals of
the polygon is
(a) 35 (b) 44 (c) 54 (d) 78

Q.3 Number of different natural numbers which are smaller than two hundred million &
using only the digits 1 or 2 is :
(a) (3) . 28  2 (b) (3) . 28  1 (c) 2 (29  1) (d) none

Q.4 5 Indian & 5 American couples meet at a party & shake hands . If no wife shakes hands
with her own husband & no Indian wife shakes hands with a male, then the number of
hand shakes that takes place in the party is :
(a) 95 (b) 110 (c) 135 (d) 150

Q.5 The number of n digit numbers which consists of the digits 1 & 2 only if each digit is to
be used atleast once, is equal to 510 then n is equal to:
(a) 7 (b) 8 (c) 9 (d) 10

Q.6 Number of six digit numbers which have 3 digits even & 3 digits odd, if each digit is to
be used atmost once is ______ .
Q.7 The tamer of wild animals has to bring one by one 5 lions & 4 tigers to the circus arena.
The number of ways this can be done if no two tigers immediately follow each other is
______.

Q.8 18 points are indicated on the perimeter of a triangle ABC (see


figure). How many triangles are there with vertices at these points?
(a) 331 (b) 408 (c) 710 (d) 711
Q.9 An English school and a Vernacular school are both under one superintendent . Suppose
that the superintendentship, the four teachership of English and Vernacular school each,
are vacant, if there be altogether 11 candidates for the appointments, 3 of whom apply
exclusively for the superintendentship and 2 exclusively for the appointment in the
English school, the number of ways in which the different appointments can be disposed
of is :
(a) 4320 (b) 268 (c) 1080 (d) 25920

Nothing is impossible, the word itself says ‘I’m possible’!


50 BINOMIAL THEOREM AND PERMUTATION & COMBINATION

Q.10 A committee of 5 is to be chosen from a group of 9 people. Number of ways in which it


can be formed if two particular persons either serve together or not at all and two other
particular persons refuse to serve with each other, is
(a) 41 (b) 36 (c) 47 (d) 76

Q.11 A question paper on mathematics consists of twelve questions divided into three parts A,
B and C, each containing four questions . In how many ways can an examinee answer
five questions, selecting atleast one from each part .
(a) 624 (b) 208 (c) 2304 (d) none
Q.12 If m denotes the number of 5 digit numbers if each successive digits are in their
descending order of magnitude and n is the corresponding figure, when the digits are in
their ascending order of magnitude then (m – n) has the value
(a) 10C4 (b) 9C5 (c) 10C3 (d) 9C3


ANSWER KEY
Q.1 (x  1)x  1 Q.2 C Q.3 A Q.4 C Q.5 C Q.6 64800
Q.7 43200 Q.8 D Q.9 D Q.10 A Q.11 A Q.12 B

ADVANCE OBJECTIVE EXERCISE – 5

Q.1 There are m points on a straight line AB & n points on the line AC none of them being
the point A. Triangles are formed with these points as vertices, when
(i) A is excluded (ii) A is included. The ratio of number of triangles in the two cases
is:
mn 2 mn 2 m n2 m (n  1)
(a) (b) (c) (d)
mn m  n 1 mn 2 (m  1) (n  1)

Q.2 Number of ways in which 7 green bottles and 8 blue bottles can be arranged in a row if
exactly 1 pair of green bottles is side by side, is (Assume all bottles to be alike except for
the colour).
(a) 84 (b) 360
(c) 504 (d) 84

Q.3 In a certain algebraical exercise book there are 4 examples on arithmetical progressions, 5
examples on permutation  combination and 6 examples on binomial theorem . Number
of ways a teacher can select for his pupils atleast one but not more than 2 examples from
each of these sets, is ______ .

CatalyseR Eduventures (India) Pvt. Ltd.


BINOMIAL THEOREM AND PERMUTATION & COMBINATION 51

Q.4 The kindergarten teacher has 25 kids in her class . She takes 5 of them at a time, to
zoological garden as often as she can, without taking the same 5 kids more than once.
Find the number of visits, the teacher makes to the garden and also the number of of
visits every kid makes.

Q.5 There are n persons and m monkeys (m > n). Number of ways in which each person may
become the owner of one monkey is
(a) nm (b) mn (c) mPn (d) mn

Q.6 Seven different coins are to be divided amongst three persons . If no two of the persons
receive the same number of coins but each receives atleast one coin & none is left over,
then the number of ways in which the division may be made is :
(a) 420 (b) 630 (c) 710 (d) none
Q.7 Let there be 9 fixed points on the circumference of a circle . Each of these points is joined
to every one of the remaining 8 points by a straight line and the points are so positioned
on the circumference that atmost 2 straight lines meet in any interior point of the circle .
The number of such interior intersection points is :
(a) 126 (b) 351 (c) 756 (d) none of these

Q.8 The number of 5 digit numbers such that the sum of their digits is even is :
(a) 50000 (b) 45000 (c) 60000 (d) none

Q.9 A forecast is to be made of the results of five cricket matches, each of which can be win,
a draw or a loss for Indian team. Find
(i) the number of different possible forecasts
(ii) the number of forecasts containing 0, 1, 2, 3, 4 and 5 errors respectively

Q.10 The number of ways in which 8 distinguishable apples can be distributed among 3 boys
such that every boy should get atleast 1 apple & atmost 4 apples is K · 7P3 where K has
the value equal to
(a) 14 (b) 66 (c) 44 (d) 22

Q.11 A women has 11 close friends. Find the number of ways in which she can invite 5 of
them to dinner, if two particular of them are not on speaking terms & will not attend
together.

Q.12 A rack has 5 different pairs of shoes. The number of ways in which 4 shoes can be
chosen from it so that there will be no complete pair is
(a) 1920 (b) 200 (c) 110 (d) 80

Nothing is impossible, the word itself says ‘I’m possible’!


52 BINOMIAL THEOREM AND PERMUTATION & COMBINATION

Paragraph for question nos. 13 to 15


Consider the word W = MISSISSIPPI

Q.13 If N denotes the number of different selections of 5 letters from the word W =
MISSISSIPPI then N belongs to the set
(a) {15, 16, 17, 18, 19} (b) {20, 21, 22, 23, 24}
(c) {25, 26, 27, 28, 29} (d) {30, 31, 32, 33, 34}

Q.14 Number of ways in which the letters of the word W can be arranged if atleast one vowel
is separated from rest of the vowels
8!·161 8 !·161 8 !·161 8 ! 165
(a) (b) (c) (d) ·
4 !· 4 !· 2 ! 4 · 4 !· 2 ! 4 !· 2 ! 4 !· 2 ! 4 !
Q.15 If the number of arrangements of the letters of the word W if all the S's and P's are
 10 ! 
separated is (K)   then K equals
 4!· 4 ! 
6 4 3
(a) (b) 1 (c) (d)
5 3 2

Q.16 In how many different ways a grandfather along with two of his grandsons and four
grand daughters can be seated in a line for a photograph so that he is always in the middle
and the two grandsons are never adjacent to each other.

ANSWER KEY
Q.1 A Q.2 C Q.3 3150 Q.4 25C 24C4 Q.5 C
5

Q.6 B Q.7 A Q.8 B Q.9 (i) 243 ; (ii) 1, 10, 40, 80, 80, 32
Q.10 D Q.11 378 Q.12 D Q.13 C Q.14 B
Q.15 B Q.16 528

CatalyseR Eduventures (India) Pvt. Ltd.


BINOMIAL THEOREM AND PERMUTATION & COMBINATION 53

ADVANCE OBJECTIVE EXERCISE – 6


Q.1 Number of different ways in which 8 different books can be distributed among 3
students, if each student receives atleast 2 books is ______.

Q.2 There are 10 seats in a double decker bus, 6 in the lower deck and 4 on the upper deck.
Ten passengers board the bus, of them 3 refuse to go to the upper deck and 2 insist on
going up. The number of ways in which the passengers can be accommodated is _____.
(Assume all seats to be duly numbered)

Q.3 Find the number of permutations of the word "AUROBIND" in which vowels appear in
an alphabetical order.

Q.4 The greatest possible number of points of intersection of 9 different straight lines & 9
different circles in a plane is:
(a) 117 (b) 153 (c) 270 (d) none

Q.5 An old man while dialing a 7 digit telephone number remembers that the first four digits
consists of one 1's, one 2's and two 3's. He also remembers that the fifth digit is either a 4
or 5 while has no memorising of the sixth digit, he remembers that the seventh digit is 9
minus the sixth digit. Maximum number of distinct trials he has to try to make sure that
he dials the correct telephone number, is
(a) 360 (b) 240 (c) 216 (d) none
Q.6 If as many more words as possible be formed out of the letters of the word
"DOGMATIC" then the number of words in which the relative order of vowels and
consonants remain unchanged is ______ .
Q.7 Number of ways in which 7 people can occupy six seats, 3 seats on each side in a first
class railway compartment if two specified persons are to be always included and occupy
adjacent seats on the same side, is (5 !) · k then k has the value equal to :
(a) 2 (b) 4 (c) 8 (d) none
Q.8 Number of ways in which 9 different toys be distributed among 4 children belonging to
different age groups in such a way that distribution among the 3 elder children is even
and the youngest one is to receive one toy more, is :
(5 !)2 9! 9!
(a) (b) (c) (d) none
8 2 3!( 2 !)3

Q.9 In an election three districts are to be canvassed by 2, 3 & 5 men respectively . If 10 men
volunteer, the number of ways they can be alloted to the different districts is :
10 ! 10 ! 10 ! 10 !
(a) (b) (c) 2
(d)
2! 3! 5! 2! 5! ( 2 !) 5 ! ( 2 !)2 3 ! 5 !

Nothing is impossible, the word itself says ‘I’m possible’!


54 BINOMIAL THEOREM AND PERMUTATION & COMBINATION

Q.10 Let Pn denotes the number of ways in which three people can be selected out of ' n '
people sitting in a row, if no two of them are consecutive. If , Pn + 1  Pn = 15 then the
value of ' n ' is :
(a) 7 (b) 8 (c) 9 (d) 10
Q.11 Number of ways in which 8 people can be arranged in a line if A and B must be next each
other and C must be somewhere behind D, is equal to
(a) 10080 (b) 5040 (c) 5050 (d) 10100
Q.12 A has 3 maps and B has 9 maps. All the 12 maps being distinct. Determine the number of
ways in which they can exchange their maps if each keeps his initial number of maps.

Q.13 Number of three digit number with atleast one 3 and at least one 2 is
(a) 58 (b) 56 (c) 54 (d) 52
Paragraph for Question Nos. 14 to 16
16 players P1, P2, P3,.......P16 take part in a tennis tournament. Lower suffix player is
better than any higher suffix player. These players are to be divided into 4 groups each
comprising of 4 players and the best from each group is selected for semifinals.
Q.14 Number of ways in which 16 players can be divided into four equal groups, is
35 8 35 8 35 8 35 8
(a)  (2r  1)
27 r 1
(b)  (2r  1)
24 r 1
(c)  (2r  1)
52 r 1
(d)  (2r  1)
6 r 1
Q.15 Number of ways in which they can be divided into 4 equal groups if the players P 1, P2, P3
and P4 are in different groups, is :
(11)! (11)! (11)! (11)!
(a) (b) (c) (d)
36 72 108 216
Q.16 Number of ways in which these 16 players can be divided into four equal groups, such
12 !
that when the best player is selected from each group, P6 is one among them, is (k) .
( 4 !)3
The value of k is :
(a) 36 (b) 24 (c) 18 (d) 20

ANSWER KEY
Q.1 2940 Q.2 172800 Q.3 8C ·4! Q.4 C Q.5 B
4

Q.6 719 Q.7 C Q.8 C Q.9 A Q.10 B

Q.11 B Q.12 219 Q.13 D Q.14 A Q.15 C

Q.16 D

CatalyseR Eduventures (India) Pvt. Ltd.


BINOMIAL THEOREM AND PERMUTATION & COMBINATION 55

ADVANCE OBJECTIVE EXERCISE – 7


Q.1 There are 10 red balls of different shades & 9 green balls of identical shades. Then the
number of arranging them in a row so that no two green balls are together is
(a) (10 !) . 11P9 (b) (10 !) . 11C9 (c) 10 ! (d) 10 ! 9 !
Q.2 Number of ways in which n distinct objects can be kept into two identical boxes so that
no box remains empty, is ______ .
Q.3 A shelf contains 20 different books of which 4 are in single volume and the others form
sets of 8, 5 and 3 volumes respectively. Number of ways in which the books may be
arranged on the shelf, if the volumes of each set are together and in their due order is
20 !
(a) (b) 7! (c) 8! (d) 7 . 8!
8 ! 5 ! 3!
Q.4 If all the letters of the word "QUEUE" are arranged in all possible manner as they are
in a dictionary, then the rank of the word QUEUE is :
(a) 15th (b) 16th (c) 17th (d) 18th
Q.5 Number of rectangles in the grid shown which are not squares is
(a) 160 (b) 162 (c) 170 (d) 185

Q.6 All the five digit numbers in which each successive digit exceeds its predecessor are
arranged in the increasing order of their magnitude. The 97th number in the list does not
contain the digit
(a) 4 (b) 5 (c) 7 (d) 8

Q.7 The number of combination of 16 things, 8 of which are alike and the rest different, taken
8 at a time is _____.

Q.8 The number of different ways in which five 'dashes' and eight 'dots' can be arranged,
using only seven of these 13 'dashes' & 'dots' is :
(a) 1287 (b) 119 (c) 120 (d) 1235520
Q.9 In a certain college at the B.Sc. examination, 3 candidates obtained first class honours in
each of the following subjects: Physics, Chemistry and Maths, no candidates obtaining
honours in more than one subject; Number of ways in which 9 scholarships of different
value be awarded to the 9 candidates if due regard is to be paid only to the places
obtained by candidates in any one subject is __________.

Q.10 There are n identical red balls & m identical green balls . The number of different linear
arrangements consisting of "n red balls but not necessarily all the green balls" is xCy then
(a) x = m + n, y = m (b) x = m + n + 1, y = m
(c) x = m + n + 1, y = m + 1 (d) x = m + n , y = n

Nothing is impossible, the word itself says ‘I’m possible’!


56 BINOMIAL THEOREM AND PERMUTATION & COMBINATION

Direction for Q.11 & Q.12


In how many ways the letters of the word “COMBINATORICS” can be arranged if
Q.11 All the vowels are always grouped together to form a contiguous block.

Q.12 All vowels and all consonants are alphabetically ordered.

Q.13 How many different arrangements are possible with the factor of the term a2 b4c5 written
at full length.

Q.14 Find the number of 4 digit numbers starting with 1 and having exactly two identical
digits.

Q.15 Number of ways in which 5 A's and 6 B's can be arranged in a row which reads the same
backwards and forwards, is

ANSWER KEY
Q.1 B Q.2 2n  1  1 Q.3 C Q.4 C
Q.5 A Q.6 B Q.7 256 Q.8 C
(9 !)(5 !) (13 !)
Q.9 1680 Q.10 B Q.11 Q.12
( 2 !)3 (8!)(5 !)
Q.13 6930 Q.14 432 Q.15 10

ADVANCE OBJECTIVE EXERCISE – 8


Q.1 Number of different words that can be formed using all the letters of the word
"DEEPMALA" if two vowels are together and the other two are also together but
separated from the first two is :
(a) 960 (b) 1200 (c) 2160 (d) 1440
Q.2 The number of ways in which 10 boys can take positions about a round table if two
particular boys must not be seated side by side is :
(a) 10 (9) ! (b) 9 (8) ! (c) 7 (8) ! (d) none
Q.3 In a unique hockey series between India & Pakistan, they decide to play on till a team
wins 5 matches . The number of ways in which the series can be won by India, if no
match ends in a draw is :
(a) 126 (b) 252 (c) 225 (d) none
Q.4 Number of cyphers at the end of 2002C is
1001

(a) 0 (b) 1 (c) 2 (d) 200

CatalyseR Eduventures (India) Pvt. Ltd.


BINOMIAL THEOREM AND PERMUTATION & COMBINATION 57

Q.5 Three vertices of a convex n sided polygon are selected. If the number of triangles that
can be constructed such that none of the sides of the triangle is also the side of the
polygon is 30, then the polygon is a
(a) Heptagon (b) Octagon (c) Nonagon (d) Decagon

Q.6 A gentleman invites a party of m + n (m  n) friends to a dinner & places m at one table
T1 and n at another table T2 , the table being round . If not all people shall have the same
neighbour in any two arrangement, then the number of ways in which he can arrange the
guests, is
(m  n) ! (m  n) ! (m  n) !
(a) (b) (c) 2 (d) none
4 mn mn mn

Q.7 There are 12 guests at a dinner party . Supposing that the master and mistress of the
house have fixed seats opposite one another, and that there are two specified guests who
must always, be placed next to one another ; the number of ways in which the company
can be placed, is:
(a) 20 . 10 ! (b) 22 . 10 ! (c) 44 . 10 ! (d) none

Q.8 Let Pn denotes the number of ways of selecting 3 people out of ' n ' sitting in a row , if
no two of them are consecutive and Qn is the corresponding figure when they are in a
circle . If Pn  Qn = 6 , then ' n ' is equal to :
(a) 8 (b) 9 (c) 10 (d) 12

Q.9 Define a 'good word' as a sequence of letters that consists only of the letters A, B and C
and in which A never immidiately followed by B, B is never immediately followed by C,
and C is never immediately followed by A. If the number of n-letter good words are 384,
find the value of n.
Q.10 Six married couple are sitting in a room. Find the number of ways in which 4 people can
be selected so that
(a) they do not form a couple (b) they form exactly one couple
(c) they form at least one couple (d) they form atmost one couple
Q.11 Fifty college teachers are surveyed as to their possession of colour TV, VCR and tape
recorder. Of them, 22 own colour TV, 15 own VCR and 14 own tape recorders. Nine of
these college teachers own exactly two items out of colour TV, VCR and tape recorders ;
and, one college teacher owns all three. how many of the 50 college teachers own none of
three, colour TV, VCR or tape recorder?
(a) 4 (b) 9 (c) 10 (d) 11

Nothing is impossible, the word itself says ‘I’m possible’!


58 BINOMIAL THEOREM AND PERMUTATION & COMBINATION

Q.12 There are counters available in x different colours. The counters are all alike except for
the colour. The total number of arrangements consisting of y counters, assuming
sufficient number of counters of each colour, if no arrangement consists of all counters of
the same colour is :
(a) xy  x (b) xy  y
(c) yx  x (d) yx  y

Q.13 There are (p + q) different books on different topics in Mathematics. (p  q)


If L = The number of ways in which these books are distributed between two students X
and Y such that X get p books and Y gets q books.
M = The number of ways in which these books are distributed between two students X
and Y such that one of them gets p books and another gets q books.
N = The number of ways in which these books are divided into two groups of p books
and q books then,
(a) L = M = N (b) L = 2M = 2N
(c) 2L = M = 2N (d) L = M = 2N

The question given below contains STATEMENT-1 (Assertion) and


STATEMENT-2 (Reason). Question has 4 choices (a) , (b) , (c) and (d) , out of
which ONLY ONE is correct. Choose the correct alternative.
Q.14 Statement 1: The sum 40C0 · 60C10 + 40C1 · 60C9 + ......... + 40C10 · 60C0 equals 100C10.
Because
Statement 2: Number of ways of selecting 10 students out of 40 boys and 60 girls is
100C .
10

(a) Statement-1 is true, statement-2 is true and statement-2 is correct explanation for
statement-1.
(b) Statement-1 is true, statement-2 is true and statement-2 is NOT the correct
explanation for statement-1.
(c) Statement-1 is true, statement-2 is false.
(d) Statement-1 is false, statement-2 is true.

CatalyseR Eduventures (India) Pvt. Ltd.


BINOMIAL THEOREM AND PERMUTATION & COMBINATION 59

MATCH THE COLUMN:


Q.15 Column-I Column-II
(A) In a plane a set of 8 parallel lines intersect a set of n parallel lines, (P) 6
that goes in another direction, forming a total of 1260 parallelograms.
The value of n is equal to (Q) 9
n1
P3 1
(B) If n
= then n is equal to (R) 10
P4 9
(C) Number of ways in which 5 persons A, B, C, D and E
can be seated on round table if A and D do not sit next (S) 12
to each other
(D) Number of cyphers at the end of the number 50P25


ANSWER KEY

Q.1 D Q.2 C Q.3 A Q.4 B Q.5 C Q.6 A

Q.7 A Q.8 C Q.9 n = 8 Q.10 240, 240, 255, 480 Q.11 C

Q.12 A Q.13 C Q.14 A Q.15 (a) R (b) Q (c) S (d) P

ADVANCE OBJECTIVE EXERCISE – 9


n 1 n
Cr
Q.1 
r 0
n
Cr  n Cr  1
is equal to

n (n  1) n1
(a) (b)
2 (n  1) 2
n(n  1) n
(c) (d)
2 2
Q.2 Let m denote the number of ways in which 4 different books are distributed among 10
persons, each receiving none or one only and let n denote the number of ways of
distribution if the books are all alike. Then :
(a) m = 4n (b) n = 4m (c) m = 24n (d) none
Q.3 The number of ways in which we can arrange n ladies & n gentlemen at a round table so
that 2 ladies or 2 gentlemen may not sit next to one another is :
(a) (n  1) ! (n  2) ! (b) (n !) (n  1) !
(c) (n + 1) ! (n) ! (d) none

Nothing is impossible, the word itself says ‘I’m possible’!


60 BINOMIAL THEOREM AND PERMUTATION & COMBINATION

Q.4 There are six periods in each working day of a school. Number of ways in which 5
subjects can be arranged if each subject is allotted at least one period and no period
remains vacant is
(a) 210 (b) 1800 (c) 360 (d) 120
Q.5 The number of all possible selections of one or more questions from 10 given questions,
each equestion having an alternative is :
(a) 310 (b) 210  1 (c) 310  1 (d) 210

Q.6 A team of 8 students goes on an excursion, in two cars, of which one can seat 5 and the
other only 4. If internal arrangement inside the car does not matter then the number of
ways in which they can travel, is
(a) 91 (b) 182 (c) 126 (d) 3920
Q.7 The number of divisors of the number 21600 is _____ and the sum of these
divisors is ______.
Q.8 10 IIT & 2 PET students sit in a row. The number of ways in which exactly 3 IIT students
sit between 2 PET student is ______ .

Q.9 The number of ways of choosing a committee of 2 women & 3 men from 5 women & 6
men, if Mr. A refuses to serve on the committee if Mr. B is a member & Mr. B can only
serve, if Miss C is the member of the committee, is :
(a) 60 (b) 84 (c) 124 (d) none

Q.10 Six persons A, B, C, D, E and F are to be seated at a circular table . The number of ways
this can be done if A must have either B or C on his right and B must have either C or D
on his right is :
(a) 36 (b) 12 (c) 24 (d) 18

Q.11 There are 2 identical white balls, 3 identical red balls and 4 green balls of different
shades. The number of ways in which they can be arranged in a row so that atleast one
ball is separated from the balls of the same colour, is :
(a) 6 (7 !  4 !) (b) 7 (6 !  4 !) (c) 8 !  5 ! (d) none

Q.12 Sameer has to make a telephone call to his friend Harish, Unfortunately he does not
remember the 7 digit phone number. But he remembers that the first three digits are 635
or 674, the number is odd and there is exactly one 9 in the number. The maximum
number of trials that Sameer has to make to be successful is
(a) 10,000 (b) 3402 (c) 3200 (d) 5000

CatalyseR Eduventures (India) Pvt. Ltd.


BINOMIAL THEOREM AND PERMUTATION & COMBINATION 61

Q.13 Six people are going to sit in a row on a bench. A and B are adjacent, C does not want to
sit adjacent to D. E and F can sit anywhere. Number of ways in which these six people
can be seated, is
(a) 200 (b) 144 (c) 120 (d) 56

Q.14 Given 11 points, of which 5 lie on one circle, other than these 5, no 4 lie on one circle .
Then the maximum number of circles that can be drawn so that each contains atleast
three of the given points is :
(a) 216 (b) 156 (c) 172 (d) none

Q.15 One hundred management students who read at least one of the three business magazines
are surveyed to study the readership pattern. It is found that 80 read Business India, 50
read Business world, and 30 read Business Today. Five students read all the three
magazines. How many read exactly two magazines?
(a) 50 (b) 10 (c) 95 (d) 25
Q.16 Find the number of 10 digit numbers using the digits 0, 1, 2, ....... 9 without repetition.
How many of these are divisible by 4.

Q.17 A four digit number is called a doublet if any of its digit is the same as only one
neighbour. For example, 1221 is a doublet but 1222 is not. Number of such doublets are
(a) 2259 (b) 2268 (c) 2277 (d) 2349


ANSWER KEY

Q.1 D Q.2 C Q.3 B Q.4 B Q.5 C Q.6 C

Q.7 72, 78120 Q.8 16 · 10! or 10C · 3! · 2! · 8! Q.9 C


3

Q.10 D Q.11 A Q.12 B Q.13 B Q.14 B Q.15 A

Q.16 (20) · 8! Q.17 B

Nothing is impossible, the word itself says ‘I’m possible’!


62 BINOMIAL THEOREM AND PERMUTATION & COMBINATION

ADVANCE OBJECTIVE EXERCISE - 10


Choose the correct alternative (only one is correct):
Q.1 There are 100 different books in a shelf. Number of ways in which 3 books can be
selected so that no two of which are neighbours is
(a) 100C3 – 98 (b) 97C3 (c) 96C3 (d) 98C3

Q.2 Two classrooms A and B having capacity of 25 and (n–25) seats respectively.An denotes
the number of possible seating arrangements of room 'A', when 'n' students are to be
seated in these rooms, starting from room 'A' which is to be filled up full to its capacity.
If An – An–1 = 25! (49C25) then 'n' equals
(a) 50 (b) 48 (c) 49 (d) 51
Q.3 The sum of all numbers greater than 1000 formed by using digits 1, 3, 5, 7 no digit being
repeated in any number is :
(a) 72215 (b) 83911 (c) 106656 (d) 114712
Q.4 Number of positive integral solutions satisfying the equation (x1 + x2 + x3) (y1 + y2) = 77,
is
(a) 150 (b) 270 (c) 420 (d) 1024
Q.5 Distinct 3 digit numbers are formed using only the digits 1, 2, 3 and 4 with each digit
used at most once in each number thus formed. The sum of all possible numbers so
formed is
(a) 6660 (b) 3330
(c) 2220 (d) none

Q.6 The streets of a city are arranged like the lines of a chess board . There are m streets
running North to South & 'n' streets running East to West . The number of ways in which
a man can travel from NW to SE corner going the shortest possible distance is :
(a) m2  n2 (b) (m  1)2 . (n  1)2
(m  n) ! (m  n  2 ) !
(c) (d)
m! . n! (m  1) ! . (n  1) !

Q.7 An ice cream parlour has ice creams in eight different varieties . Number of ways of
choosing 3 ice creams taking atleast two ice creams of the same variety, is :
(a) 56 (b) 64 (c) 100 (d) none
(Assume that ice creams of the same variety are identical & available in unlimited
supply)

CatalyseR Eduventures (India) Pvt. Ltd.


BINOMIAL THEOREM AND PERMUTATION & COMBINATION 63

Q.8 There are 12 books on Algebra and Calculus in our library , the books of the same subject
being different. If the number of selections each of which consists of 3 books on each
topic is greatest then the number of books of Algebra and Calculus in the library are
respectively:
(a) 3 and 9 (b) 4 and 8 (c) 5 and 7 (d) 6 and 6
Q.9 The sum of all the numbers formed from the digits 1, 3, 5, 7, 9 which are smaller than
10,000 if repetion of digits is not allowed, is
(a) (28011)S (b) (28041)S (c) (28121)S (d) (29152)S
where S = (1+3+5+7+9)
Choose the correct alternatives (More than one are correct):
Q.10 The combinatorial coefficient C(n, r) is equal to
(a) number of possible subsets of r members from a set of n distinct members.
(b) number of possible binary messages of length n with exactly r 1's.
(c) number of non decreasing 2-D paths from the lattice point (0, 0) to (r, n).
(d) number of ways of selecting r things out of n different things when a particular
thing is always included plus the number of ways of selecting 'r' things out of n,
when a particular thing is always excluded.

Q.11 Identify the correct statement(s).


(a) Number of naughts standing at the end of is 30 .
(b) A telegraph has 10 arms and each arm is capable of 9 distinct positions excluding
the position of rest. The number of signals that can be transmitted is 1010  1 .
(c) Number of numbers greater than 4 lacs which can be formed by using only the
digits 0, 2, 2, 4, 4 and 5 is 90.
(d) In a table tennis tournament, every player plays with every other player. If the
number of games played is 5050 then the number of players in the tournament is
100.
Q.12 There are 10 questions, each question is either True or False. Number of different
sequences of incorrect answers is also equal to
(a) Number of ways in which a normal coin tossed 10 times would fall in a definite
order if both Heads and Tails are present.
(b) Number of ways in which a multiple choice question containing 10 alternatives
with one or more than one correct alternatives, can be answered.
(c) Number of ways in which it is possible to draw a sum of money with 10 coins of
different denominations taken some or all at a time.
(d) Number of different selections of 10 indistinguishable things taken some or all at
a time.

Nothing is impossible, the word itself says ‘I’m possible’!


64 BINOMIAL THEOREM AND PERMUTATION & COMBINATION

Q.13 The continued product, 2 . 6 . 10 . 14 ...... to n factors is equal to


(a) 2nCn (b) 2nPn
(c) (n + 1) (n + 2) (n + 3) ...... (n + n) (d) none
Q.14 The Number of ways in which five different books to be distributed among 3 persons so
that each person gets at least one book, is equal to the number of ways in which
(a) 5 persons are allotted 3 different residential flats so that and each person is alloted
at most one flat and no two persons are alloted the same flat.
(b) number of parallelograms (some of which may be overlapping) formed by one set
of 6 parallel lines and other set of 5 parallel lines that goes in other direction.
(c) 5 different toys are to be distributed among 3 children, so that each child gets at
least one toy.
(d) 3 mathematics professors are assigned five different lecturers to be delivered , so
that each professor gets at least one lecturer.

Q.15 The combinatorial coefficient n – 1Cp denotes


(a) the number of ways in which n things of which p are alike and rest different can be
arranged in a circle.
(b) the number of ways in which p different things can be selected out of n different
thing if a particular thing is always excluded.
(c) number of ways in which n alike balls can be distributed in p different boxes so that
no box remains empty and each box can hold any number of balls.
(d) the number of ways in which (n – 2) white balls and p black balls can be arranged in
a line if black balls are separated, balls are all alike except for the colour.
Q.16 The maximum number of permutations of 2n letters in which there are only a's & b's,
taken all at a time is given by :
2nC 2 6 10 4n  6 4n  2
(a) n (b) . . ...... .
1 2 3 n 1 n
n 1 n  2 n  3 n  4 2n  1 2n
(c) . . . ...... .
1 2 3 4 n 1 n
2n . 1 . 3 . 5 ...... ( 2 n  3) (2 n  1)
(d)
n!
Q.17 Number of ways in which 3 numbers in A.P. can be selected from 1, 2, 3, ...... n is :
 n 1
2
n  n  2
(a)   if n is even (b) if n is odd
 2  4
2
n 1 n  n  2
(c) if n is odd (d) if n is even
4 4

CatalyseR Eduventures (India) Pvt. Ltd.


BINOMIAL THEOREM AND PERMUTATION & COMBINATION 65

Q.18 If P(n, n) denotes the number of permutations of n different things taken all at a time
then P(n, n) is also identical to
(a) r! · P(n, n – r) (b) (n – r) · P(n, r)
(c) n · P(n – 1, n – 1) (d) P(n, n – 1)
where 0  r  n

Q.19 Which of the following statements are correct?


(a) Number of words that can be formed with 6 only of the letters of the word
"CENTRIFUGAL" if each word must contain all the vowels is 3 · 7!
(b) There are 15 balls of which some are white and the rest black. If the number of ways
in which the balls can be arranged in a row, is maximum then the number of white
balls must be equal to 7 or 8. Assume balls of the same colour to be alike.
(c) There are 12 things, 4 alike of one kind, 5 alike and of another kind and the rest are
all different. The total number of combinations is 240.
(d) Number of selections that can be made of 6 letters from the word "COMMITTEE" is
35.

MATCH THE COLUMN:


Q.20 Column I Column II
(a) Number of increasing permutations of m symbols are there (P) nm
from the n set numbers {a1, a2, , an} where the
order among the numbers is given by a1 < a2 < a3 <  an–1 < an is
(b) There are m men and n monkeys. Number of ways in which (Q) mC
n
every monkey has a master, if a man can have any number of
monkeys
(c) Number of ways in which n red balls
and (m – 1) green balls can be arranged (R) nC
m
in a line, so that no two red balls are together, is
(balls of the same colour are alike)
(d) Number of ways in which 'm' different
toys can be distributed in 'n' children (S) mn
if every child may receive any number of toys, is

Nothing is impossible, the word itself says ‘I’m possible’!


66 BINOMIAL THEOREM AND PERMUTATION & COMBINATION

Q.21 Column-I Column-II


(a) Four different movies are running in a town. Ten students (P) 11
go to watch these four movies. The number of ways in which
every movie is watched by atleast one student, is (Assume
each way differs only by number of students watching a
movie)
(b) Consider 8 vertices of a regular octagon and its centre. If T (Q) 36
denotes the number of triangles and S denotes the number of
straight lines that can be formed with these 9 points then the
value of (T – S) equals
(c) In an examination, 5 children were found to have their mobiles (R) 52
in their pocket. The Invigilator fired them and took their mobiles
in his possession. Towards the end of the test, Invigilator randomly
returned their mobiles. The number of ways in which at most
two children did not get their own mobiles is
(d) The product of the digits of 3214 is 24. The number of 4 digit (S) 60
natural numbers such that the product of their digits is 12, is
(E) The number of ways in which a mixed double tennis game
can be arranged from amongst 5 married couple if no
husband & wife plays in the same game, is (T) 84

SUBJECTIVE:
Q.22 A commitee of 10 members is to be formed with members chosen from the faculties of
Arts, Economics, Education, Engineering, Medicine and Science. Number of possible
ways in which the faculties representation be distributed on this committee, is ________.
(Assume every department contains more than 10 members).

Q.23 How many ways are there to seat n married couples (n  3) around a table such that men
and women alternate and each women is not adjacent to her husband.

Q.24 10 identical ball are distributed in 5 different boxes kept in a row and labled A, B, C, D
and E. Find the number of ways in which the ball can be distributed in the boxes if no
two adjacent boxes remain empty.

Q.25 The number of non negative integral solution of the inequation x + y + z + w  7 is


____.

CatalyseR Eduventures (India) Pvt. Ltd.


BINOMIAL THEOREM AND PERMUTATION & COMBINATION 67

Q.26 On the normal chess board as shown, I1 & I2 are two insects which starts moving
towards each other. Each insect moving with the same constant speed. Insect I 1 can
move only to the right or upward along the lines while the insect I2 can move only to the
left or downward along the lines of the chess board. Prove that the total number of
ways the two insects can meet at same point during their trip is equal to
 9   10   11   12   13   14   15   16 
8  7   6   5   4   3   2   1 
               
OR
 1   3   5   7   9   11   13   15 
28                
1  2   3   4   5   6   7   8 
OR
 2   6   10   14   18   22   26   30 
1 2  3   4   5   6   7   8 
               
Q.27 How many numbers gretater than 1000 can be formed from the digits 112340 taken 4 at a
time.
Q.28 Tom has 15 ping-pong balls each uniquely numbered from 1 to 15. He also has a red box,
a blue box, and a green box.
(a) How many ways can Tom place the 15 distinct balls into the three boxes so that
no box is empty?
(b) Suppose now that Tom has placed 5 ping-pong balls in each box. How many
ways can he choose 5 balls from the three boxes so that he chooses at least one
from each box?
Q.29 Find the number of ways in which 12 identical coins can be distributed in 6 different
purses, if not more than 3 & not less than 1 coin goes in each purse.

Q.30 A drawer is fitted with n compartments and each compartment contains n counter, no
two of which marked alike. Number of combinations which can be made with these
counters if no two out of the same compartment enter into any combination, is ______ .

Q.31 In how many ways it is possible to select six letters, including at least one vowel from the
letters of the word "F L A B E L L I F O R M ". (It is a picnic spot in
U.S.A.)


Nothing is impossible, the word itself says ‘I’m possible’!


68 BINOMIAL THEOREM AND PERMUTATION & COMBINATION

ANSWER KEY

Q.1 D Q.2 A Q.3 C Q.4 C Q.5 A Q.6 D

Q.7 B Q.8 D Q.9 B Q.10 A, B, D Q.11 B, C

Q.12 B, C Q.13 B, C Q.14 B,C,D Q.15 B, D

Q.16 A, B, C, D Q.17 C, D Q.18 A, C, D Q.19 A, B, D

Q.20 (a) R; (b) S; (c) Q; (d) P Q.21 (a) T; (b) R; (c) P; (d) Q; (E) S

Q.22 3003 Q.23 n!(n – 1)! – 2(n – 1)! Q.24 771 ways Q.25 330

Q.26 12870 Q.27 159 Q.28 (a) 315 – 3 · 215 + 3; (b) 2250

Q.29 141 Q.30 (n + 1)n  1 Q.31 296

CatalyseR Eduventures (India) Pvt. Ltd.


BINOMIAL THEOREM AND PERMUTATION & COMBINATION 69

2.4. ADVANCE SUBJECTIVE EXERCISES:

ADVANCE SUBJECTIVE EXERCISE – 1


Q.1 The straight lines l1 , l2 & l3 are parallel & lie in the same plane. A total of m points are
taken on the line l1 , n points on l2 & k points on l3. How many maximum number of
triangles are there whose vertices are at these points ?

Q.2 (a) How many five digits numbers divisible by 3 can be formed using the digits 0, 1,
2, 3, 4, 7 and 8 if each digit is to be used atmost once.
(b) Find the number of 4 digit positive integers if the product of their digits is
divisible by 3.
Q.3 There are 2 women participating in a chess tournament. Every participant played 2 games
with the other participants. The number of games that the men played between
themselves exceeded by 66 as compared to the number of games that the men played
with the women. Find the number of participants & the total numbers of games played in
the tournament.
Q.4 All the 7 digit numbers containing each of the digits 1, 2, 3, 4, 5, 6, 7 exactly once, and
not divisible by 5 are arranged in the increasing order. Find the (2004) th number in this
list.
Q.5 5 boys & 4 girls sit in a straight line. Find the number of ways in which they can be
seated if 2 girls are together & the other 2 are also together but separate from the first 2.

Q.6 A crew of an eight oar boat has to be chosen out of 11 men five of whom can row on
stroke side only, four on the bow side only, and the remaining two on either side. How
many different selections can be made?
Q.7 An examination paper consists of 12 questions divided into parts A & B.
Part-A contains 7 questions & PartB contains 5 questions. A candidate is required to
attempt 8 questions selecting atleast 3 from each part. In how many maximum ways can
the candidate select the questions ?

Q.8 In how many ways can a team of 6 horses be selected out of a stud of 16 , so that there
shall always be 3 out of A B C A  B  C  , but never A A  , B B  or C C  together.

Q.9 During a draw of lottery, tickets bearing numbers 1, 2, 3,......, 40, 6 tickets are drawn out
& then arranged in the descending order of their numbers. In how many ways, it is
possible to have 4th ticket bearing number 25.

Nothing is impossible, the word itself says ‘I’m possible’!


70 BINOMIAL THEOREM AND PERMUTATION & COMBINATION

Q.10 Find the number of distinct natural numbers upto a maximum of 4 digits and divisible by
5, which can be formed with the digits 0, 1, 2, 3, 4, 5, 6, 7, 8, 9 each digit not occuring
more than once in each number.

Q.11 The Indian cricket team with eleven players, the team manager, the physiotherapist and
two umpires are to travel from the hotel where they are staying to the stadium where the
test match is to be played. Four of them residing in the same town own cars, each a four
seater which they will drive themselves. The bus which was to pick them up failed to
arrive in time after leaving the opposite team at the stadium. In how many ways can they
be seated in the cars ? In how many ways can they travel by these cars so as to reach in
time, if the seating arrangement in each car is immaterial and all the cars reach the
stadium by the same route.
Q.12 There are n straight lines in a plane, no 2 of which parallel , & no 3 pass through the same
point. Their point of intersection are joined. Show that the number of maximum fresh
n(n  1)(n  2 )(n  3)
lines thus introduced is .
8

Q.13 In how many ways can you divide a pack of 52 cards equally among 4 players. In how
many ways the cards can be divided in 4 sets, 3 of them having 17 cards each & the 4th
with 1 card.

Q.14 A firm of Chartered Accountants in Bombay has to send 10 clerks to 5 different


companies, two clerks in each. Two of the companies are in Bombay and the others are
outside. Two of the clerks prefer to work in Bombay while three others prefer to work
outside. In how many ways can the assignment be made if the preferences are to be
satisfied.
Q.15 A train going from Cambridge to London stops at nine intermediate stations. 6 persons
enter the train during the journey with 6 different tickets of the same class. How many
different sets of ticket may they have had?

Q.16 How many arrangements each consisting of 2 vowels & 2 consonants can be made out of
the letters of the word ‘DEVASTATION’?

Q.17 Find the number of ways in which the letters of the word 'KUTKUT' can be arranged so
that no two alike letters are together.

Q.18 Find the number of words each consisting of 3 consonants & 3 vowels that can be formed
from the letters of the word “Circumference”. In how many of these c’s will be together.

CatalyseR Eduventures (India) Pvt. Ltd.


BINOMIAL THEOREM AND PERMUTATION & COMBINATION 71

Q.19 There are 5 white , 4 yellow , 3 green , 2 blue & 1 red ball. The balls are all identical
except for colour. These are to be arranged in a line in 5 places. Find the number of
distinct arrangements.
Q.20 How many 4 digit numbers are there which contains not more than 2 different digits?

Q.21 An 8 oared boat is to be manned by a crew chosen from 14 men of which 4 can only steer
but can not row & the rest can row but cannot steer. Of those who can row, 2 can row on
the bow side. In how many ways can the crew be arranged.
Q.22 (a) A flight of stairs has 10 steps. A person can go up the steps one at a time, two at a
time, or any combination of 1's and 2's. Find the total number of ways in which
the person can go up the stairs.
(b) You walk up 12 steps, going up either 1 or 2 steps with each stride. There is a
snake on the 8th step, so you can not step there. Number of ways you can go up.

Q.23 Each of 3 committees has 1 vacancy which is to be filled from a group of 6 people. Find
the number of ways the 3 vacancies can be filled if ;
(i) Each person can serve on atmost 1 committee.
(ii) There is no restriction on the number of committees on which a person can serve.
(iii) Each person can serve on atmost 2 committees.

Q.24 How many ten digit whole numbers satisfy the following property they have 2 and 5 as
digits, and there are no consecutive 2's in the number (i.e. any two 2's are separated by at
least one 5).

Q.25 In how many other ways can the letters of the word MULTIPLE be arranged;
(i) without changing the order of the vowels
(ii) keeping the position of each vowel fixed &
(iii) without changing the relative order/position of vowels & consonants.

Q.26 12 persons are to be seated at a square table, three on each side. 2 persons wish to sit on
the north side and two wish to sit on the east side. One other person insists on occupying
the middle seat (which may be on any side). Find the number of ways they can be seated.
Q.27 How many integers between 1000 and 9999 have exactly one pair of equal digit such as
4049 or 9902 but not 4449 or 4040?
Q.28 Determine the number of paths from the origin to the point (9, 9) in the cartesian plane
which never pass through (5, 5) in paths consisting only of steps going 1 unit North and 1
unit East.
Q.29 (i) Prove that : nPr = n1Pr + r. n1Pr1 (ii) If 20Cr+2 = 20C2r3 find 12Cr
(iii) Prove that n1C + n1C4 > nC3 if n > 7. (iv) Find r if 15C3r = 15Cr+3
3

Nothing is impossible, the word itself says ‘I’m possible’!


72 BINOMIAL THEOREM AND PERMUTATION & COMBINATION

Q.30 There are 20 books on Algebra & Calculus in our library. Prove that the greatest number
of selections each of which consists of 5 books on each topic is possible only when there
are 10 books on each topic in the library.

ANSWER KEY
Q.1 m+n+kC  (mC3 + nC3 + kC3) Q.2 (a) 744 (b) 7704 Q.3 13, 156
3

Q.4 4316527 Q.5 43200 Q.6 145

Q.7 420 Q.8 960 Q.9 24C . 15C3


2

11! . 4! 52 ! 52 !
Q.10 1106 Q.11 12! ; Q.13 ;
(3!)4 2! (13!) 4
3!(17 !)3

Q.14 5400 Q.15 45C Q.16 1638


6

Q.17 30 Q.18 22100 , 52 Q.19 2111

Q.20 576 Q.21 4 . (4!)2 . 8C4 . 6C2 Q.22 (a) 89, (b) 63

Q.23 120, 216, 210 Q.24 143

Q.25 (i) 3359 (ii) 59 (iii) 359 Q.26 2! 3! 8! Q.27 3888

Q.28 30980 Q.29 (ii) 792 ; (iv) r = 3

CatalyseR Eduventures (India) Pvt. Ltd.


BINOMIAL THEOREM AND PERMUTATION & COMBINATION 73

ADVANCE SUBJECTIVE EXERCISE – 2


Q.1 How many 6 digits odd numbers greater than 60,0000 can be formed from the digits 5, 6,
7, 8, 9, 0 if (i) repetitions are not allowed (ii) repetitions are allowed.
Q.2 A man has 3 friends. In how many ways he can invite one friend everyday for dinner on 6
successive nights so that no friend is invited more than 3 times.

Q.3 Find the number of 7 lettered words each consisting of 3 vowels and 4 consonants which
can be formed using the letters of the word "DIFFERENTIATION".

Q.4 Find the number of ways in which 3 distinct numbers can be selected from the set
{31, 32, 33, ....... 3100, 3101} so that they form a G.P.
Q.5 There are 2n guests at a dinner party. Supposing that the master and mistress of the house
have fixed seats opposite one another, and that there are two specified guests who must
not be placed next to one another. Show that the number of ways in which the company
can be placed is (2n  2)!.(4n2  6n + 4).

Q.6 The members of a chess club took part in a round robin competition in which each plays
every one else once. All members scored the same number of points, except four juniors
whose total score were 17.5. How many members were there in the club? Assume that for
each win a player scores 1 point , for draw 1/2 point and zero for losing.

Q.7 In an election for the managing committee of a reputed club , the number of candidates
contesting elections exceeds the number of members to be elected by r (r > 0). If a voter
can vote in 967 different ways to elect the managing committee by voting atleast 1 of
them & can vote in 55 different ways to elect (r  1) candidates by voting in the same
manner. Find the number of candidates contesting the elections & the number of
candidates losing the elections.

INSTRUCTION FOR QUESTION NOS. 8 TO 10:


2 Americal men; 2 British men; 2 Chinese men and one each of Dutch, Egyptial, French
and German persons are to be seated for a round table conference.
Q.8 If the number of ways in which they can be seated if exactly two pairs of persons of same
nationality are together is p(6!), then find p.

Q.9 If the number of ways in which only American pair is adjacent is equal to q(6!), then find
q.

Q.10 If the number of ways in which no two people of the same nationality are together given
by r (6!), find r.

Nothing is impossible, the word itself says ‘I’m possible’!


74 BINOMIAL THEOREM AND PERMUTATION & COMBINATION

Q.11 For each positive integer k, let Sk denote the increasing arithmetic sequence of integers
whose first term is 1 and whose common difference is k. For example, S3 is the sequence
1, 4, 7, 10...... Find the number of values of k for which Sk contain the term 361.

Q.12 A shop sells 6 different flavours of ice-cream. In how many ways can a customer choose
4 ice-cream cones if
(i) they are all of different flavours
(ii) they are non necessarily of different flavours
(iii) they contain only 3 different flavours
(iv) they contain only 2 or 3 different flavours?

Q.13 There are n triangles of positive area that have one vertex A(0, 0) and the other two
vertices whose coordinates are drawn independently with replacement from the set
{0, 1, 2, 3, 4} e.g. (1, 2), (0, 1), (2, 2) etc. Find the value of n.

Q.14 (a) How many divisors are there of the number x = 21600. Find also the sum of these
divisors.
(b) In how many ways the number 7056 can be resolved as a product of 2 factors.
(c) Find the number of ways in which the number 300300 can be split into 2 factors
which are relatively prime.
(d) Find the number of positive integers that are divisors of atleast one of the numbers
1010 ; 157 ; 1811.

Q.15 How many 15 letter arrangements of 5 A's, 5 B's and 5 C's have no A's in the first 5
letters, no B's in the next 5 letters, and no C's in the last 5 letters.
Q.16 How many different ways can 15 Candy bars be distributed between Ram, Shyam,
Ghanshyam and Balram, if Ram can not have more than 5 candy bars and Shyam must
have at least two. Assume all Candy bars to be alike.

Q.17 Find the number of three digits numbers from 100 to 999 inclusive which have any one
digit that is the average of the other two.

Q.18 Find the number of distinct throws which can be thrown with 'n' six faced normal dice
which are indistinguishable among themselves.
Q.19 There are 15 rowing clubs; two of the clubs have each 3 boats on the river; five others
have each 2 and the remaining eight have each 1; find the number of ways in which a list
can be formed of the order of the 24 boats, observing that the second boat of a club
cannot be above the first and the third above the second. How many ways are there in
which a boat of the club having single boat on the river is at the third place in the list
formed above?

CatalyseR Eduventures (India) Pvt. Ltd.


BINOMIAL THEOREM AND PERMUTATION & COMBINATION 75

Q.20 Consider a 7 digit telephone number 336 - 7624 which has the property that the first three
digit prefix, 336 equals the product of the last four digits. How many seven digit phone
numbers beginning with 336 have this property, e.g. (336–7624)

Q.21 Find the sum of all numbers greater than 10000 formed by using the digits 0 , 1 , 2 , 4 , 5
no digit being repeated in any number.
Q.22 There are 3 cars of different make available to transport 3 girls and 5 boys on a field trip.
Each car can hold up to 3 children. Find
(a) the number of ways in which they can be accomodated.
(b) the numbers of ways in which they can be accomodated if 2 or 3 girls are assigned
to one of the cars.In both the cases internal arrangement of children inside the car
is considered to be immaterial.
Q.23 Find the number of three elements sets of positive integers {a, b, c} such that a × b × c =
2310.

Q.24 Find the number of integer between 1 and 10000 with at least one 8 and at least one 9 as
digits.

Q.25 In IndoPak one day International cricket match at Sharjah , India needs 14 runs to win
just before the start of the final over. Find the number of ways in which India just
manages to win the match (i.e. scores exactly 14 runs) , assuming that all the runs are
made off the bat & the batsman can not score more than 4 runs off any ball.

Q.26 A man goes in for an examination in which there are 4 papers with a maximum of m
marks for each paper; show that the number of ways of getting 2m marks on the whole is
1
(m + 1) (2m2 + 4m + 3).
3

ANSWER KEY
Q.1 240 , 15552 Q.2 510 Q.3 532770 Q.4 2500
Q.6 27 Q.7 10, 3 Q.8 60 Q.9 64
Q.10 244 Q.11 24 Q.12 (i) 15, (ii) 126, (iii) 60, (iv) 105
Q.13 256 Q.14 (a) 72; 78120; (b) 23; (c) 32 ; (d) 435
Q.15 2252 Q.16 440 Q.17 121 Q.18 n + 5C5
24 ! 23!
Q.19 2 5
; 8C1 . Q.20 84 Q.21 3119976
(3!) ( 2!) (3!)2 ( 2!)5
Q.22 (a) 1680; (b) 1140 Q.23 40 Q.24 974 Q.25 1506

Nothing is impossible, the word itself says ‘I’m possible’!


76 BINOMIAL THEOREM AND PERMUTATION & COMBINATION

JEE (MAIN) CORNER

1
1. If the expansion in powers of x of the function is
1– ax 1– bx 
a0 + a1x + a2x2 + a3x3 + ......., then an is : (2006)
an – bn a n 1 – b n 1 b n 1 – a n 1 bn – a n
(a) (b) (c) (d)
b–a b–a b–a b–a

2. For natural numbers m, n if (1 – y)m (1 + y)n= 1 + a1y + a2y2 + ..... and a1 = a2 = 10,
then (m, n) is : (2006)
(a) (35, 20) (b) (45, 35)
(c) (35, 45) (d) (20, 45)
3. The set S : = {1, 2, 3 ..........12} is to be partitioned into three sets A, B, C of equal size.
Thus, AU B U C = S, A  B = B  C = A  C = 
The number of ways to partition S is - (2007)
(a) 12!/3! (4!)3 (b) 12!/3!(3!)4
(c) 12!/(4!)3 (d) 12!/(3!)4

4. The sum of the series 20C0 – 20C1 + 20C2 – 20C3 + ..... + 20C10 is (2007)
1 20
(a) –20C10 (b) C10
2
(c) 0 (d) 20C10

5. In a shop there are five types of ice-creams available.. A child buys six ice-creams.
Statement-1 : The number of different ways the child can buy the six ice-creams, is
10C . (2008)
5

Statement-2 : The number of different ways the child can buy the six ice-creams is equal
to the number of different ways of arranging 6A' s and 4 B' s in a row.
(a) Statement-1 is True, Statement-2 is True; Statement-2 is a correct explanation for
Statement-1.
(b) Statement-1 is True, Statement-2 is True; Statement-2 is NOT a correct
explanation for Statement-1
(c) Statement-1 is True, Statement-2 is False
(d) Statement-1 is False, Statement-2 is True

CatalyseR Eduventures (India) Pvt. Ltd.


BINOMIAL THEOREM AND PERMUTATION & COMBINATION 77

6. How many different words can be formed by jumbling the letters in the word
MISSISSIPPI in which no two S are adjacent ? (2008)
(a) 8. 6C4 . 7C4 (b) 6. 7 8C4 (c) 6. 8. 7C4 (d) 7. 6C4 . 8C4
n
7. STATEMENT-1 :  ( r  1) nCr = (n + 2) 2n–1 (2008)
r0

n
STATEMENT-2 :  (r + 1) nCr xr = (1 + x)n + nx (1 + x)n – 1
r 0

(a) Statement-1 is True, Statement-2 is True; Statement-2 is a correct explanation for


Statement-1.
(b) Statement-1 is True, Statement-2 is True; Statement-2 is NOT a correct
explanation for Statement-1
(c) Statement-1 is True, Statement-2 is False
(d) Statement-1 is False, Statement-2 is True

8. From 6 different novels and 3 different dictionaries, 4 novels and 1 dictionary are to be
selected and arranged in a row on the shelf so that the dictionary is always in the middle.
Then the number of such arrangements is- (2009)
(a) atleast 500 but less than 750 (b) atleast 750 but less than 10000
(c) atleast 1000 (d) less than 500
10 10 10
9. Let S1 =  j ( j –1) 10Cj , S2 =  j
j 1 j 1
10C and S =
j 3 j
j 1
2 10C .
j (2010)

STATEMENT -1 : S3 = 55 × 29 .

STATEMENT -2 : S1 = 90 × 28 and S2 = 10 × 28.


(a) Statement -1 is true, Statement-2 is true ; Statement -2 is not a correct
explanation for Statement -1.
(b) Statement-1 is true, Statement-2 is false.
(c) Statement -1 is false, Statement -2 is true.
(d) Statement -1 is true, Statement -2 is true; Statement-2 is a correct explanation for
Statement-1.
6
10. The coefficient of x 7 in the expansion of 1  x  x 2  x 3  is (2011)
(a) – 132 (b) 144
(c) 132 (d) 144

Nothing is impossible, the word itself says ‘I’m possible’!


78 BINOMIAL THEOREM AND PERMUTATION & COMBINATION

7
11. Statement – 1: For each natural number n,  n  1  1 is divisible by 7.

Statement – 2 : For each natural number n,n7  n is divisible by 7. (2011)


(a) Statement – 1 is true, Statement – 2 is true; Statement – 2 is a correct explanation
for Statement – 1.
(b) Statement – 1 is ture, Statemnet -2 is true; Statement – 2 is NOT a correct
explanation for Statement -1
(c) Statement – 1 is ture, Statement -2 is false
(d) Statement -1 is false, Statement -2 is true
2n 2n
12. If n is a positive integer , then   
3 1  3 1  is: (2012)

(a) an irrational number


(b) an odd positive integer
(c) an even positive integer
(d) a rational number other than positive integers
10
 x 1 x 1 
13. The term independent of x in expansion of  2/3 1/3
 1/2 
is (2013)
 x  x 1 x  x 
(a) 4 (b) 120
(c) 210 (d) 310
18
14. 
If the coefficients of x 3 and x 4 in the expansion of 1  ax  bx 2 1  2x  in powers of
x are both zero, then (a, b) is equal to (2014)
 251   251   272   272 
(a) 16, (b) 14, (c)  14, (d)  16,
 3   3   3   3 

15. The number of ways of selecting 15 teams from 15 men and 15 women, such that each
team consists of a man and a woman, is : (2015)
(a) 1120 (b) 1240 (c) 1880 (d) 1960

n
16. If the coefficients of the three successive terms in the binomial expansion of 1  x  are
in the ratio 1 : 7 : 42, then the first of these terms in the expansion is : (2015)
(a) 6th (b) 7th (c) 8th (d) 9th

17. If all the words (with or without meaning) having five letters, formed using the letters of
the word SMALL and arranged as in a dictionary; then the position of the word SMALL
is : (2016)
th th th nd
(a) 58 (b) 46 (c) 59 (d) 52
CatalyseR Eduventures (India) Pvt. Ltd.
BINOMIAL THEOREM AND PERMUTATION & COMBINATION 79

n
 2 4
18. If the number of terms in the expansion of  1   2  , x  0, is 28, then the sum of the
 x x 
coefficients of all the terms in this expansion, is : (2016)
(a) 729 (b) 64 (c) 2187 (d) 243

19. The value of


 21
C1 10 C1    21 C2 10 C2    21 C3 10 C3    21 C4 10 C4   .....   21 C10 10 C10  is:
(2017)
(a) 221  211 (b) 221  210 (c) 220  29 (d) 220  210

20. A man X has 7 friends, 4 of them are ladies and 3 are men. His wife Y also has 7 friends,
3 of them are ladies and 4 are men. Assume X and Y have no common friends. Then the
total number of ways in which X and Y together can throw a party inviting 3 ladies and 3
men, so that 3 friends of each of X and Y are in this party, is: (2017)
(a) 485 (b) 468 (c) 469 (d) 484



ANSWER KEY

1 2 3 4 5 6 7 8 9 10 11 12 13 14
C C C B D D A C B B A A C D
15 16 17 18 19 20
B B A A D A

Nothing is impossible, the word itself says ‘I’m possible’!


80 BINOMIAL THEOREM AND PERMUTATION & COMBINATION

JEE (ADVANCED) CORNER:

1. If r, s, t are prime numbers and p, q are the positive integers such that their LCM of p, q is
is r2t4s2, then the numbers of ordered pair of (p, q) is (2006)
(a) 252 (b) 254 (c) 225 (d) 224

2. The letters of the word COCHIN are permuted and all the permutations are arranged in
an alphabetical order as in an English dictionary. The number of words that appear before
the word COCHIN is (2007)
(a) 360 (b) 192 (c) 96 (d) 48

3. The number of seven digit integers, with sum of the digits equal to 10 and formed by
using the digits 1,2 and 3 only, is (2009)
(a) 55 (b) 66 (c) 77 (d) 88

4. For r = 0,1, …., 10, let A r ,Br and Cr denote, respectively, the coefficient of x r in the
10
10 20 30
expansions of 1  x  , 1  x  and 1  x  . Then  A B
r 1
r Br  C10 A r  is equal to
10

(2010)
(a) B10  C10 (b) A 10  B C10 A10  (c) 0
2
10 (d) C10  B10

5. The total number of ways in which 5 balls of different colours can be distributed among 3
persoons so that each person gets at least one ball is (2012)
(a) 75 (b) 150 (c) 210 (d) 9

Passage – 1
Let a n denote the number of all n-digit positive integers formed by the digits 0,1 or both
such that no consecutive digits in them are 0. Let bn = the number of such n-digit
integers ending with digit 1 and c n = the number of such n-digit integers ending with
digit 0.

6. The value of b6 is (2012)


(a) 7 (b) 8 (c) 9 (d) 11

7. Which of the following is correct? (2012)


(a) a17  a16  a15 (b) c17  c 16  c15
(c) b17  b16  c16 (d) a17  c17  b16

CatalyseR Eduventures (India) Pvt. Ltd.


BINOMIAL THEOREM AND PERMUTATION & COMBINATION 81

8. Consider the set of eight vectors V = ai  b j  ck : a, b, c  {  1,1} . Three non-coplanar
 
vectors can be chosen from V in 2p ways. Then p is (2013)

n 5
9. The coefficients of three consecutive terms of 1  x  are in the ratio 5 : 10 : 14. Then
n= (2013)

4 7 12
10. Cofficient of x11 in the expansion of 1  x 2   1  x  1  x 
3 4
is (2014)
(a) 1051 (b) 1106 (c) 1113 (d) 1120

11. Six cards and six envelopes are numbered 1, 2, 3, 4, 5, 6 and cards are to be placed in
envelopes so that each envelope contains exactly one card and no card is placed in the
envelope bearing the same number and moreover the card numbered 1 is always placed in
envelope numbered 2. Then the number of ways it can be done is (2014)
(a) 264 (b) 265
(c) 53 (d) 67
12. Let n be the number of ways in which 5 boys and 5 girls can stand in a queue in such a
way that all the girls stand consecutively in the queue. Let m be the number of ways in
which 5 boys and 5 girls can stand in a queue in such a way that exactly four girls stand
m
consecutively in the queue. Then the value of is - (2015)
n

13. The coefficient of x9 in the expansion of 1  x  1  x 2 1  x 3  ..... 1  x100  is - (2015)

14. A debate club consists of 6 girls and 4 boys. A team of 4 members is to be selected from
this club including the selection of a captain (from among these 4 members) for the team.
If the team has to include at most one boy. Then the number of ways of selecting the
team is (2016)
(a) 380 (b) 320
(c) 260 (d) 95

15. Let m be the smallest positive integer such that the coefficient of x 2 in the expansion of
2 3 49 50 51
1  x   1  x   ......  1  x   1  mx  is  3n  1 C3
for some positive integer n. Then the value of n is - (2016)

Nothing is impossible, the word itself says ‘I’m possible’!


82 BINOMIAL THEOREM AND PERMUTATION & COMBINATION

16. Words of length 10 are formed using the letters A, B, C, D, E, F, G, H, I, J. Let x be the
number of such words where no letter is repeated; and let y be the number of such words
y
where exactly one letter is repeated twice and no other letter is repeated. Then, 
9x
(2017)

17. Let S  1, 2,3,......,9 . For k  1, 2,.....,5, let N k be the number of subsets of S, each
containing five elements out of which exactly k are odd. Then N1  N 2  N 3  N 4  N 5  ?
(2017)
(a) 210 (b) 252 (c) 125 (d) 126


ANSWER KEY
1 2 3 4 5 6 7 8 9 10 11 12 13 14 15 16 17
C C C D B B A 5 6 C C 2 8 A 5 5 D

CatalyseR Eduventures (India) Pvt. Ltd.

You might also like